Егэ 2018 химия изменения: Изменения ЕГЭ 2018 по химии: официальные изменения от ФИПИ

Содержание

ЕГЭ 2018 по химии – HIMI4KA

Этот курс предназначен прежде всего для школьников старших классов для подготовки к единому государственному экзамену по химии. Я бы характеризовал его как уроки для прагматиков, желающих получить на ЕГЭ 2018 максимальный балл.

В уроках в сжатой, справочной форме изложен теоретический материал в соответствии с Федеральным компонентом государственных стандартов основного общего и среднего (полного) общего образования и обязательным минимумом содержания среднего (полного) общего образования по химии. Кроме того, рассмотрены спецификация контрольных измерительных материалов и кодификатор элементов содержания и требования к уровню подготовки выпускников общеобразовательных учреждений для единого государственного экзамена по химии.

После теоретического материала отдельно разбираются наиболее сложные задания повышенного уровня сложности с кратким ответом и задания высокого уровня сложности с развернутым ответом. В курсе подготовки к ЕГЭ также приведены задания повышенного уровня сложности с кратким ответом и задания высокого уровня сложности с развернутым ответом. Выполнение этих заданий поможет оценить уровень ваших знаний, определить, какие темы следует повторить.

Данный курс претерпел изменения по сравнению с курсом подготовки 2015–2017 годов как в части теоретического материала, так и в заданиях повышенного уровня сложности с кратким ответом. Практика показывает, что задания именно этой категории являются достаточно сложными для выпускников. Отдельно рассмотрены окислительно-восстановительные реакции с участием органических веществ (составная часть задания высокого уровня сложности с развернутым ответом). Максимальное количество баллов на экзамене можно получить, правильно ответив на задания высокого уровня сложности с развернутым ответом и на задания повышенного уровня сложности с кратким ответом. Для этого нужно твердо знать химические свойства важнейших химических элементов, а также основных классов неорганических и органических веществ. Теоретического материала, изложенного в уроках, для этого вполне достаточно.

Хочется подчеркнуть, что данное пособие не подменяет существующие учебники и учебные пособия (в первую очередь рекомендованные Рособрнадзором и ФИПИ), а лишь дополняет их, поэтому наряду с данным курсом рекомендуем изучить самоучитель и видео-уроки на сайте HIMI4KA.RU.

Если ты готов, то дерзай! И да прибудет с тобой химия 🙂

Самоучитель

Оценка

Варианты заданий ЕГЭ 2018 по химии / Ю.Н. Медведев

Уважаемые выпускники и абитуриенты!

Настоящее учебное пособие представляет собой сборник заданий для подготовки к сдаче Единого государственного экзамена (ЕГЭ) по химии, который является как выпускным экзаменом за курс средней школы, так и вступительным экзаменом в вуз. Структура пособия отражает современные требования к процедуре сдачи ЕГЭ по химии, что позволит вам лучше подготовиться к новым формам выпускной аттестации и к поступлению в вузы.

Пособие состоит из 10 вариантов заданий, которые по форме и содержанию приближены к демоверсии ЕГЭ и не выходят за рамки содержания курса химии, нормативно определенного Федеральным компонентом государственного стандарта общего образования. Химия (приказ Минобразования № 1089 от 05.03.2004 г.).

Уровень предъявления содержания учебного материала в заданиях соотнесен с требованиями государственного стандарта к подготовке выпускников средней (полной) школы по химии.

В контрольных измерительных’ материалах Единого государственного экзамена используются задания трех типов:

—  задания базового уровня сложности с кратким ответом,

—  задания повышенного уровня сложности с кратким ответом,

—  задания высокого уровня сложности с развернутым ответом.

Каждый вариант экзаменационной работы построен по единому плану. Работа состоит из двух частей, включающих в себя суммарно 34 задания. Часть 1 содержит 29 заданий с кратким ответом базового и повышенного уровня сложности. Часть 2 содержит 5 заданий высокого уровня сложности, с развернутым ответом (задания под номерами 30-34).

В заданиях высокого уровня сложности текст решения записывается на специальном бланке. Задания именно этого типа составляют основную часть письменной работы по химии на вступительных экзаменах в вузы.

Задания с развернутым ответом могут быть выполнены выпускниками разными способами. Поэтому решения, приведенные в методических рекомендациях для экзаменаторов (имеются в виду критерии оценивания заданий части 2 в разделе «Решение заданий варианта 1»), следует рассматривать как один из возможных вариантов ответов.

Назначение данного пособия — ознакомить читателей со структурой контрольных измерительных материалов, числом, формой и уровнем сложности заданий. Эти сведения позволят выпускникам выработать стратегию подготовки и сдачи ЕГЭ в соответствии с целями, которые они ставят перед собой.

В пособии даны ответы к заданиям всех вариантов и приведены подробные решения всех заданий десятого варианта. Кроме того, приведены образцы бланков, используемых на ЕГЭ для записи ответов и решений. Именно такой вид имеют контрольные измерительные материалы, которые получают выпускники на экзамене. Прежде чем приступить к решению заданий, изучите внимательно все инструкции.

Настоящее пособие адресовано учащимся-старшеклассникам и абитуриентам для самоподготовки и самоконтроля. Пособие может быть использовано учителями химии и методистами для подготовки учащихся к итоговой аттестации по химии за курс средней школы, причем как в форме ЕГЭ, так и традиционного письменного экзамена.

Я сдам ЕГЭ! ЕГЭ 2018. Химия. Каверина, Медведев, Молчанова и др. In-chemistry.ru

Предлагаем Вашему вниманию учебное пособие «Я сдам ЕГЭ! Химия. Курс самоподготовки. Технология решения заданий». Данное пособие создано авторами из числа членов Федеральной комиссии по разработке контрольных измерительных материалов ЕГЭ. Оно предназначено для быстрой и эффективной подготовки учащихся 10—11 классов к Единому государственному экзамену. Пособие содержит весь необходимый материал, изучение которого является залогом успешной сдачи ЕГЭ. В пособии представлены не только задачи, но и теоретический материал для их решения, представленный в доступной и наглядной форме в виде графиков, иллюстраций, схем и таблиц.

СОДЕРЖАНИЕ
Предисловие 3
Тематическое планирование курса «Я СДАМ ЕГЭ» 4

МОДУЛЬ I. ТЕОРЕТИЧЕСКИЕ ОСНОВЫ ХИМИИ 7
Занятие 1. Строение электронных оболочек и электронная конфигу¬рация атомов. Периодический закон и Периодическая система химических элементов Д. И. Менделеева. Закономерности изменения химических свойств элементов и их соединений по периодам и группам —
Занятие 2. Электроотрицательность, степень окисления и валентность химических элементов. Химическая связь. Вещества молекулярного и немолекулярного строения 17
Занятие 3. Классификация и номенклатура неорганических веществ. Классификация химических реакций 30
Занятие 4. Скорость реакции, её зависимость от различных факторов.
Обратимые и необратимые химические реакции. Химическое равновесие 41
Занятие 5. Электролитическая диссоциация. Сильные и слабые электролиты. Реакции ионного обмена 46
Занятие 6. Гидролиз 51
Занятие 7. Окислительно-восстановительные реакции 55
МОДУЛЬ II. НЕОРГАНИЧЕСКАЯ ХИМИЯ 71
Занятие 8. Характерные химические свойства представителей основных классов неорганических веществ —
Занятие 9. Свойства галогенов и их соединений 83
Занятие 10. Неметаллы VIA-группы: кислород, сера 94
Занятие 11. Азот и его соединения 108
Занятие 12. Фосфор и его соединения 120
Занятие 13. Неметаллы IVA-группы: углерод, кремний 128
Занятие 14. Общие свойства металлов. Металлы А-групп 140
Занятие 15. Металлы В-групп 145
МОДУЛЬ III. ОРГАНИЧЕСКАЯ ХИМИЯ 159
Занятие 16. Классификация и номенклатура органических веществ. Теория строения органических соединений —
Занятие 17. Алканы и циклоалканы 165
Занятие 18. Алкены и алкадиены 169
Занятие 19. Алкины 174
Занятие 20. Ароматические углеводороды 177
Занятие 21. Спирты и фенолы 188
Занятие 22. Карбонильные соединения: альдегиды и кетоны 197
Занятие 23. Карбоновые кислоты и сложные эфиры 202
Занятие 24. Амины и аминокислоты 216
Занятие 25. Жиры. Углеводы 224
МОДУЛЬ IV. КОЛИЧЕСТВЕННЫЕ ОТНОШЕНИЯ В ХИМИИ: РАСЧЁТЫ ПО ХИМИЧЕСКИМ ФОРМУЛАМ И УРАВНЕНИЯМ ХИМИЧЕСКИХ РЕАКЦИЙ 234
Занятие 26. Расчёты массы вещества или объёма газа по известному количеству вещества, массе или объёму одного из участвующих в реакции веществ 235
Занятие 27. Расчёты с использованием понятия «массовая доля вещества в растворе» 238
Занятие 28. Задачи на установление молекулярной и структурной формулы вещества 246

Скачать “471- ЕГЭ-2018. Химия. Я сдам. Курс самопод._Каверина_2018 -256с” 471-EGE`-2018.-KHimiya.-YA-sdam.-Kurs-samopod._Kaverina_2018-256s.pdf – Загружено 5478 раз – 56 МБ

Добавлено в: Документы Метки: ЕГЭ

Тесты ЕГЭ по химии 2021

Тесты ЕГЭ по химии

Об экзамене

Химию нельзя сдать на высокий балл, просто выучив теорию. Химию надо любить. И любить всем сердцем, чтобы тот безграничный объем информации ровным слоем уложился в голове. А когда мы говорим о будущем, о профессиях, которые будут востребованы долгие годы, то практически все направления, которые связаны с химией, с рынка труда никуда не уйдут. Потребность в “химических мозгах” стабильно растет. Поэтому если вы чувствуете, что есть хоть какая-то предрасположенность к данной науке, не поленитесь и попробуйте себя. И если все получится, то вам откроется безграничное поле для новых открытий и свершений. Меняйте себя и мир вокруг!

Структура

Часть 1 содержит 29 заданий с кратким ответом, часть 2 содержит 5 заданий высокого уровня сложности, с развернутым ответом (порядковые номера этих заданий: 30, 31, 32, 33, 34). На выполнение всех заданий отводится 3,5 часа.

Дополнительные материалы и оборудование

К каждому варианту экзаменационной работы прилагаются следующие материалы:

− Периодическая система химических элементов Д.И. Менделеева;

− таблица растворимости солей, кислот и оснований в воде;

− электрохимический ряд напряжений металлов.

Во время выполнения экзаменационной работы разрешается использовать непрограммируемый калькулятор.

Пояснения к оцениванию заданий

За правильный ответ на каждое из заданий 1–8, 12–16, 20, 21, 27–29 ставится 1 балл. Задание считается выполненным верно, если экзаменуемый дал правильный ответ в виде последовательности цифр или числа с заданной степенью точности.

Задания 9–11, 17–19, 22–26 считаются выполненными верно, если правильно указана последовательность цифр. За полный правильный ответ в заданиях 9–11, 17–19, 22–26 ставится 2 балла; если допущена одна ошибка, – 1 балл; за неверный ответ (более одной ошибки) или его отсутствие – 0 баллов.

Задания части 2 (с развёрнутым ответом) предусматривают проверку от трёх до пяти элементов ответа. Задания с развёрнутым ответом могут быть выполнены выпускниками различными способами. Наличие каждого требуемого элемента ответа оценивается 1 баллом, поэтому максимальная оценка верно выполненного задания составляет от 3 до 5 баллов в зависимости от степени сложности задания: задание 30 – 3 балла; 31 – 4 балла; 32 – 5 баллов; 33 – 4 балла; 34 – 4 балла. Проверка заданий части 2 осуществляется на основе сравнения ответа выпускника с поэлементным анализом приведённого образца ответа.

Любой учитель или репетитор может отслеживать результаты своих учеников по всей группе или классу. Для этого нажмите ниже на кнопку «Создать класс», а затем отправьте приглашение всем заинтересованным.

Ознакомьтесь с подробной видеоинструкцией по использованию модуля.


Более 700 сахалинских выпускников написали ЕГЭ по химии и истории. Сахалин.Инфо

16:52 4 июня 2018

Образование, Южно-Сахалинск

Сегодня выпускники Сахалинской области приступили к сдаче единого государственного экзамена по химии и истории. На экзамен по истории пришли 453 человека. Химию писали 276 выпускников. Данные предметы одиннадцатиклассники пишут по выбору.

Результаты ЕГЭ по химии нужны выпускникам для поступления в вузы на обучение по направлениям: «Химия», «Фармакология», «Здравоохранение», «Химическая технология», «Экология и природопользование», «ОБЖ», «Педагогика» (учитель химии) и другие.

Минимальный балл ЕГЭ по химии, ниже которого вузы не могут устанавливать проходной порог для абитуриентов, составляет 36 тестовых баллов. На выполнение экзаменационной работы отводится 3,5 часа. Экзаменационная работа включает 35 заданий.

Единый государственный экзамен по истории необходим для поступления в вузы на такие специальности и направления подготовки, как «История», «Политология», «Международные отношения», «Туризм», «Искусства и гуманитарные науки», «История искусств», «Социальная работа» и другие.

Минимальный балл ЕГЭ по истории — 32. На выполнение экзаменационной работы отводится 3 часа 55 минут. Каждый вариант экзаменационной работы по истории состоит из двух частей и включает в себя 25 заданий, различающихся формой и уровнем сложности. Структура и содержание экзаменационной работы 2018 года не претерпели изменений по сравнению с моделью 2017 года. Свои результаты ЕГЭ по химии и истории одиннадцатиклассники узнают не позднее 20 июня.

— Хочу отметить, что проведение экзамена на высоком организационном уровне, а также усиление ответственности всех лиц, привлекаемых к проведению ЕГЭ — это одни из основных задач по обеспечению качественного проведения основного периода государственной итоговой аттестации в Сахалинской области, — подчеркнула первый заместитель министра образования Сахалинской области Светлана Мирова.

Основной период ЕГЭ-2018 продлится до 2 июля. В период государственной итоговой аттестации действует региональная горячая линия. По всем вопросам организации и проведения ЕГЭ можно обращаться по телефонам: 8(4242) 46-59-60, 46-59-93.

В период ЕГЭ действует также горячая линия Рособрнадзора: 8(495) 984-89-19 и телефон доверия ЕГЭ: 8(495) 104-68-38, сообщает ИА Sakh.com со ссылкой на министерство образования области.

Анна Костромина и Дмитрий Добротин: Как подготовиться к госэкзамену по химии | Программа: Информационная программа «ОТРажение» | ОТР

Дмитрий Добротин

руководитель Федеральной комиссии разработчиков КИМ ЕГЭ и ОГЭ по химии

Оксана Галькевич: Юра, ты так быстро «откланял» Тимура Уразбахтина, а я хотела его спросить, любил ли он химию в школе. Ну хорошо, спрошу тебя. Ты как?

Юрий Коваленко: Я любил, это один из моих любимых предметов, на самом деле.

Оксана Галькевич: Любил, да? Органическую, неорганическую?

Юрий Коваленко: Органическую, я за естественность.

Оксана Галькевич: Ну давай, сейчас будешь отдуваться за всю российскую журналистику. Уважаемые друзья, это наша рубрика «О ЕГЭ предметно». Сегодня будем говорить о едином государственном экзамене по химии. Расскажем о том, как экзамен проходит, дадим некие рекомендации – не мы, конечно, а наши гости, эксперты в студии – как к экзамену готовиться лучше, и расскажем о некоторых критериях, как оцениваются знания учащегося.

Юрий Коваленко: Специалист Федерального института педагогических измерений даст совет, что можно взять на экзамен, чем на нем пользоваться и как самостоятельно оценить свои знания, где их можно проверить. Давайте тогда и разбираться.

Оксана Галькевич: Но прежде, может быть, справочку дадим по экзамену, именно по ЕГЭ по химии. Дело в том, что вы все знаете, уважаемые друзья, что есть экзамены обязательные, а есть экзамены по выбору. Так вот химия – это предмет по выбору. Оценка по этому предмету необходима для поступления на химический, химико-технологический, медицинский, биологический факультеты, а также при поступлении на специальности, которые связаны с организацией общественного питания, например, строительством, легкой промышленностью. Поэтому экзамен этот сейчас очень популярен, насколько популярен, буквально через секунду-другую выясним у наших гостей. В прошлом году его в нашей стране сдавали около 74 тысяч человек. Для успешной сдачи экзамена необходимо набрать 36 тестовых баллов, но в прошлом году эту планку не смогли, к сожалению, преодолеть 15% сдававших экзамен. При этом средний балл на экзамене был равен 55 баллам.

Юрий Коваленко: Экзамен длится 3.5 часа, за это время необходимо сделать несколько заданий, о которых сейчас, собственно, речь и пойдет.

А говорим мы сегодня с нашими гостями: у нас в студии Дмитрий Добротин, руководитель Федеральной комиссии разработчиков КИМ ЕГЭ и ОГЭ по химии, и Анна Костромина, учитель химии московской школы №1028. Здравствуйте.

Оксана Галькевич: Здравствуйте, уважаемые гости, Дмитрий Юрьевич, Анна Ивановна.

Анна Костромина: Здравствуйте.

Дмитрий Добротин: Здравствуйте.

Оксана Галькевич: Уважаемые телезритель, подключайтесь, пожалуйста, к нашей беседе. Если вдруг вы молодой человек и учитесь сейчас в 9-11-х классах – заранее ведь ребята выбирают себе профессию, специальность – пожалуйста, звоните нам, задавайте вопросы, потому что у нас в студии человек, который непосредственно занимается подготовкой этих самых тестовых заданий ЕГЭ по химии, уважаемые друзья. Если вы педагог, пожалуйста, звоните, пообщаетесь с Анной Ивановной, которая расскажет вам, как правильно готовить ребят к экзамену. Если вы родитель и тоже сопереживаете, тоже звоните и тоже о чем-нибудь спрашивайте нас. Уважаемые друзья, пишите, звоните нам, на SMS-портал присылайте ваши сообщения – 3443, первые буквы «ОТР».

Скажите, пожалуйста, мы сказали о том, что сейчас это очень популярный, распространенный экзамен, 74 тысячи сдавали только в прошлом году. Какая прибавка по сравнению с 2016-2015-и гг.? – динамика какая?

Дмитрий Добротин: Честно говоря, динамика практически никакая. То есть у нас стабильный контингент тех, кто выбирает наш экзамен. То есть за последние…

Оксана Галькевич: Сложный просто экзамен, на самом деле.

Дмитрий Добротин: Да, в том числе, наверное, по причине его сложности, наверное, по причине его конкретности и достаточно все-таки высоких требований. Вот та цифра, которую вы назвали, 15% все-таки не преодолевают эту планку, как раз отражает, что там есть нам чем подумать, а значит, есть то, над чем задуматься при подготовке и системно построить подготовку к экзамену.

Юрий Коваленко: А вот достаточно ли школьной программы, для того чтобы подготовиться к ЕГЭ по химии, или все-таки необходимо брать репетитора? Потому что 15% уже говорят о том, что каждый 7-й не может сдать этот экзамен.

Дмитрий Добротин: Здесь, наверное, надо сказать, что очень важный момент, на каком уровне в школе изучается химия. И очевидно, что если курс изучается в объеме 1 час в неделю, конечно, предполагается какая-то дополнительная подготовка, и это необязательно репетитор, это может быть просто очень хорошее взаимодействие с учителем, но тогда большая нагрузка ложится на самого ученика, самостоятельность он должен проявить. Если же это классы с углубленным изучением, то в принципе, как правило, подготовки и времени на подготовку хватает в том числе без привлечения репетиторов. В этом плане как раз Анна Ивановна абсолютно точно может поделиться своим опытом, потому что очень многое зависит и от личности учителя, и от того, как он умеет спланировать подготовку ребенка. Поэтому здесь это такой момент взаимодействия, выстраивания взаимодействия учителя и ученика, если мало часов. Если больше часов, там легче подготовка, точно успевают. Наверное, минимум 3 часа необходимы для…

Оксана Галькевич: А «мало часов» – это вы имеете в виду обычную школу…

Анна Костромина: Да, это базовый уровень.

Оксана Галькевич: А побольше – это специализированное, углубленное изучение, да?

Анна Костромина: Да, профильный уровень.

Оксана Галькевич: Вы в такой школе преподаете, в таком классе, или вы все-таки в обычной общеобразовательной?

Анна Костромина: У меня есть базовый уровень преподавания химии и профильный уровень преподавания химии в 10-11-х классах.

Оксана Галькевич: И вот как ребята, которые выбирают этот предмет для сдачи в качестве единого государственного экзамена… Вы просто даете им какие-то дополнительные задания? Вы их организуете вокруг себя на какую-то дополнительную работу, остаетесь после уроков? Как вы выстраиваете это взаимодействие, о котором говорит Дмитрий Юрьевич?

Анна Костромина: Существует программа, по которой мы обучаем, существуют у нас кодификаторы, спецификации, которые позволяют нам детей готовить к единому государственному экзамену. Но все обучение построено как раз-таки не для того, чтобы подготовить ребенка именно решать тестовые задания, а научить ребенка мыслить. Потому что если в КИМе произошло изменение хотя бы малейшее формулировок, ребенок растеряется, если мы не научим его мыслить. Поэтому прежде всего учим мыслить, рассуждать логически.

Юрий Коваленко: То есть одних формул недостаточно, необходимо какое-то творчество и пространственное мышление, чтобы понять, как все это..?

Анна Костромина: Нет, одних формул никогда не было достаточно.

Дмитрий Добротин: Я здесь просто хочу добавить, что, конечно, чем больше вообще вариантов заданий не только в тестовой форме – ОГЭ, ЕГЭ – а чем более разнообразны, чем больше ребенок тренируется, тем более спокойно он выходит на экзамен. Поэтому вот эта подготовка, конечно, не должна строиться только в направлении заданий, которые включатся в контрольно-измерительные материалы. То есть здесь вариативность в плане заданий и в плане уровня этих заданий, конечно, самому ребенку показывает, что он может, он справляется, и на этапе непосредственно написания варианта это, конечно, придает ему уверенность и спокойствие, а это очень важно на экзамене.

Оксана Галькевич: Дмитрий Юрьевич, вы как человек, который, собственно, и формулировками этих самых заданий занимается, вы стараетесь позаковыристее как-нибудь, посложнее, или наоборот, ничего лишнего, чтобы было понятно?

Дмитрий Добротин: Вы знаете, тут зависит от формы задания. Потому что есть задания, которые должны непосредственно выводить ребенка на очень конкретный ответ при всем при том, что он, конечно, должен размышлять. То есть у нас практически нет заданий на репродукцию «знаешь или не знаешь», то есть все равно какой-то процесс размышления предполагается. Есть задания, которые подводят ребенка к необходимости выстраивать какие-то логические цепочки, причинно-следственные связи; есть задания, которые предполагают создание некоего модельного эксперимента, когда ребенок должен по условию задания воспроизвести в своем образном мышлении те процессы, которые протекают с веществами, то, как они изменяются в процессе протекания химических реакций. Поэтому создание образов и работа, может быть, на формирование образов… Учителя, конечно, этим занимаются, в том числе, конечно, этому должен способствовать химический эксперимент, который все-таки во многом отличает наш курс от многих других.

Оксана Галькевич: Давайте об эксперименте еще тоже поговорим. Давайте послушаем нашу телезрительницу Татьяну из Нижнего Тагила. Татьяна, здравствуйте.

Зритель: Здравствуйте.

Оксана Галькевич: Здравствуйте. Вы мама или педагог?

Зритель: Я мама того, кто в прошлом году сдавал экзамен по химии в 11 классе, и педагог тоже, но не по химии. Хотелось бы задать вопрос тем, кто создает эти тесты ЕГЭ: почему учебник у них базовый уровень, на нем написано так, и урок один-единственный в неделю, о каких экспериментах можно на нем говорить? Почему хотят большего от них? Конечно, мы за свой счет, так сказать, проходили все, что…

Оксана Галькевич: С репетиторами то есть, да?

Зритель: …за пределами учебника и, конечно же, нарешивание задач гораздо сложнее, чем в учебнике.

Оксана Галькевич: Да, спасибо, Татьяна. Это, наверное, и вам тоже вопрос.

Анна Костромина: Это как раз то, о чем я и говорила. Если вы уже определились заранее, что ребенок идет поступать в ВУЗ, где требуется экзамен по химии, значит, уже после 9 класса нужно четко определиться, что нужно химию изучать на профильном или углубленном уровне. Потому что, конечно, базовый уровень не даст нам той возможности, того большого объема информации, который нужен ребенку, и это все ложится дополнительно на плечи ребенка. То есть больший объем информации придется ребенку самому осваивать, больше проявлять самостоятельности. Хотя, опять-таки, педагог рядышком, педагог может направить, дать определенные задания и проверить.

Оксана Галькевич: А педагог готов? Попался у вас такой, знаете, очень любознательный, но без репетитора, и вот пристает постоянно с вопросами: «Дайте задание». Педагог, ваши коллеги сейчас готовы уделять внимание такому дотошному?

Анна Костромина: Я думаю, что да. Такие дети всегда очень интересны, очень интересны.

Оксана Галькевич: Вы знаете, как будто это счастье. На самом деле мне кажется, что да, это…

Анна Костромина: Для учителя это счастье, когда такие дети.

Юрий Коваленко: Интерес в глазах школьника.

А все же какие из заданий считаются заведомо легкими, а какие чаще всего проваливают на экзамене?

Анна Костромина: Достаточно сложными являются расчетные задачи. Это традиционно сложно, потому что включают в себя они не только нашу химическую науку, но и математическую составляющую, от этого никуда не уйдешь.

Юрий Коваленко: То есть если плохо с математикой, с химией будет тоже беда?

Анна Костромина: Ну не могу сказать, что дети, сдающие химию, имеют проблемы с математикой, но иногда случаются казусы расчетного плана. То есть нужно в комбинации все это. Поэтому, как правило, детям сложнее вот именно расчетные задания даются.

Юрий Коваленко: А легче?

Анна Костромина: А легче – это первая часть наша, традиционная, где достаточно даже бывает и базового уровня знаний, если ребенок трудится самостоятельно.

Юрий Коваленко: Ну к примеру, какой-нибудь вопрос из первой части.

Анна Костромина: Я думаю, что самые простые для детей вопросы – это по периодической системе химических элементов.

Оксана Галькевич: У-у-у, это, мне кажется, даже мы с Юрой сейчас…

Юрий Коваленко: Это «Назовите 48-й элемент» или что?

Анна Костромина: Нет, почему? Элементы даны, то есть перечислены элементы, и их нужно расположить в определенной последовательности.

Дмитрий Добротин: Например, в порядке возрастания радиусов атомов.

Анна Костромина: Да.

Дмитрий Добротин: Достаточно легко дети справляются с классификационными заданиями, на классификацию – это установление соответствия между формулой вещества и классом, группой, к которой он принадлежит, дети достаточно успешно с этим справляются. Наверное, это в том числе и потому, что начинают эти задания отрабатывать еще на этапе – как раз то, о чем говорила Анна Ивановна – 8-9-х классов, потом это только совершенствуется. И есть некоторый материал, который прирастает уже в 10-11-х классах, в том числе органика, например, органическая химия начинается в 10 классе. Конечно, вопросы в этом плане, по этому разделу тоже нередко вызывают, может быть, чуть большие затруднения; может быть, это связано со спецификой записи формул органических веществ, которые нередко записываются в развернутом виде, и для детей с, может быть, недостаточно развитым образным мышлением это вызывает дополнительные трудности. Поэтому такие трудности тоже бывают в этом плане.

Оксана Галькевич: Два звонка в очереди, принимаем?

Дмитрий Добротин: Принимаем, конечно.

Оксана Галькевич: Из Петербурга тоже Татьяна, здравствуйте.

Зритель: Из Петербурга я, Екатерина Юрьевна.

Оксана Галькевич: Здравствуйте, прошу прощения, неправильно мне сказали.

Зритель: Здравствуйте.

Оксана Галькевич: Слушаем вас.

Зритель: Я сама заканчивала Институт Герцена по специализации «Химия», но, к сожалению, не смогла работать в школе, но оба сына у меня сдавали ЕГЭ волевым решением мамы. И вы знаете, я хочу сказать, что на самом деле…

Я согласна с Анной Ивановной, что очень важна именно логика в химии, то есть на самом деле в ЕГЭ очень много заданий, связанных с исключениями из общих правил. На самом деле действительно дети не знают их и не должны, может быть, знать. Может быть, действительно больше обратить внимания на логику химии? На самом деле очень жалко, что ушли из программы учебники Рудзитиса и Фельдмана, на самом деле они сейчас занимаются в основном по Габриеляну, там очень… Там не всегда выдержана химическая логика. Поэтому мне кажется, что на это тоже стоит обратить внимание в школах при подготовке к ЕГЭ и ОГЭ, чтобы вернулись все-таки к классике. Спасибо.

Оксана Галькевич: Спасибо. Квалифицированное мнение, авторитетное мнение, опыт вашего коллеги.

Дмитрий Добротин: Да.

Анна Костромина: Да, более чем.

Дмитрий Добротин: Наверное, можно отчасти согласиться, что учебники играют действительно большую роль, и то, насколько логично изложен сам материал учебника и то, насколько учителю удается эти логические умения сформировать, причем, опять же, не только на заданиях, но и на вопросах, которые предполагают полные развернутые ответы. То есть опять же вот это умение ребенка формулировать полно и развернуто, мне кажется, и по жизни очень важное умение, но когда ребенок еще и говорит на научном языке с использованием химической терминологии, это тоже очень здорово развивает его мышление. Поэтому…

А что касается качества учебников, то есть определенные проблемы. Но здесь надо что, наверное, сказать? Сколько людей, столько и мнений. Потому что я знаю, что, например, есть достаточно большая группа учителей, которые с симпатией относятся к учебнику Олега Сергеевича Габриеляна, в то же время, например, имеют претензии к учебнику других авторов, которые были названы – Рудзитис, Фельдман.

Оксана Галькевич: Кстати говоря, давайте попросим наших режиссеров показать сайт ФИПИ, на котором, уважаемые друзья, есть открытый банк заданий единого государственного экзамена за предыдущие годы. Вот вы открываете страницу ФИПИ, находите этот открытый банк заданий, кликаете, сейчас он у меня откроется, выбираете здесь предмет, который вы намерены подготовить для единого госэкзамена (в данном случае сегодня у нас химия) – тут открывается, смотрите: «Теоретические основы химии», «Неорганическая химия», «Органическая химия»…

Дмитрий Добротин: Разделы основные.

Оксана Галькевич: Разделы, да. В органическую мне даже страшно заглядывать, какие там вопросы.

Я хотела спросить Анну Ивановну: скажите, это в принципе хороший метод подготовки к единому госкзамену – прорешивать просто задания за предыдущие годы?

Анна Костромина: Да, это очень удобно.

Оксана Галькевич: Очень удобно.

Анна Костромина: И детям удобно.

Оксана Галькевич: То есть это можно в любой точке нашей страны делать? Не у всех есть возможность, может быть, с репетиторами дополнительно заниматься. Если человек, так скажем, настроен на какую-то задачу, на победу, на решение подтянуть себя по химии, неплохо подготовиться к экзамену, может быть, даже и поступить…

Дмитрий Добротин: Да-да, конечно. Так и надо рассчитывать, что обязательно поступишь. И здесь банк все-таки, вы знаете, как одна из составляющих подготовки. То есть если только замыкаться на заданиях банка, то абсолютно согласен с Анной Ивановной, что есть ситуация, что вариативность этих заданий может поставить в тупик, что «Я решал вот такие задания, встретилось немножко другое», ребенок начинает теряться. Поэтому здесь надо более разнообразные задания решать и на каком-то этапе переходить уже на решение вариантов, чтобы понимать, как планировать время на решение, может быть, даже в каком порядке решать какие части.

Оксана Галькевич: А вот как? Расскажите, в каком порядке решать какие части. С чего надо начинать? – наверное, с самого простого?

Дмитрий Добротин: Да. Вообще психологи рекомендуют действительно идти от простого к сложному, и если задание вызывает какие-то очень серьезные трудности, пропускать его и идти дальше. Я знаю достаточно большое количество детей-выпускников, которые начинают как раз с более сложной части, потому что там ты вынужден записывать все подробно, таким образом, ты сразу погружаешься в мир именно знаков и образов, а потом, как бы активизировав свой мозг, ты переходишь уже к тем заданиям, которые предполагают более, скажем так, возможность выбора из предложенных вариантов, возможность опоры на оставленные формулы.

Анна Костромина: Более простые действия.

Дмитрий Добротин: Да, более простые действия.

Анна Костромина: Сколько детей, столько будет методов.

Оксана Галькевич: Вы не настаиваете, никогда не говорите, что «Вот только вот так делать, начинаем с простого и дальше к сложному?»

Анна Костромина: А нет смысла. Ребенка нельзя ломать, он должен продумать, как он будет делать…

Оксана Галькевич: Редко услышишь от нашего педагога такое.

Анна Костромина: Наша работа заключается в том, чтобы направлять, то есть предложить варианты, так попробовать и так попробовать, ребенок должен выбрать. То есть я считаю, что это очень важно, чтобы ребенок выбрал.

Юрий Коваленко: Есть ли какие-то подвохи на экзамене? Все-таки химия – это такой достаточно практический предмет. Есть ли какой-то практический опыт во время экзамена, или это исключено?

Дмитрий Добротин: Именно проведение опытов в ЕГЭ не предполагается, в ОГЭ есть модель, которая предполагает проведение эксперимента. Если мы говорим про ЕГЭ, то тем не менее есть задания, выполнение которых очень точно опирается на умение проводить эксперимент и на знание, например, признаков протекания химических реакций.

Юрий Коваленко: То есть качественная реакция какая-то.

Дмитрий Добротин: Таких заданий порядка 3-4 как минимум в едином государственном экзамене точно есть, поэтому экспериментальная составляющая играет важную роль.

Оксана Галькевич: Ирина из Москвы, здравствуйте.

Зритель: Здравствуйте. Вот я тоже мама, мы готовимся второй раз уже проходить ЕГЭ. Старший ребенок у меня уже проходил ЕГЭ, мы очень успешно сдали и поступили во все ВУЗы. Но вот хотелось бы сейчас пожелание такое составителям программ ЕГЭ. У нас же сейчас принят федеральный образовательный государственный стандарт в образовании и учебники, и вот хотелось бы, чтобы все эти программы, которые составляются для ЕГЭ, были в пределах федерального образовательного государственного стандарта были, чтобы не загружать ребенка занятиями с репетиторами. Потому что как бы ни для кого не секрет: чтобы успешно сдать ЕГЭ, нужно заниматься с репетитором, но это ложится большая нагрузка на ребенка. Мы, конечно, все успеваем, все сдаем, делаем, но как бы вот подумать: ребенок учится, 11 класс в основном 7 уроков, и представляете, после 7 уроков он еще бежит куда-то к репетиторам. Это практически каждый день, потому что 4 экзамена надо сдать, по 4-м предметам мы готовимся. Мы сдали все успешно, но это была очень большая нагрузка для ребенка.

Оксана Галькевич: И для родителей тоже, да.

Зритель: Хотелось бы чтобы все эти составления ЕГЭ были в пределах школьной программы, это мое пожелание как мамы, которая как бы все это видела.

Оксана Галькевич: Да, Ирина, спасибо большое. Очень много сообщений, кстати говоря, на нашем SMS-портале – спрашивают, почему вопросы по органике из профиля. «Почему в ЕГЭ встречаются ошибки?», – пишет нам Белгородская область (не знаю, на чем основано это мнение). «Почему задания открытого сегмента ФИПИ не систематизированы по химии элементов и нет ответов?», – спрашивает Волгоградская область. Если можно, очень коротко.

Дмитрий Добротин: В принципе ответы там должны быть, потому что это действительно тренировочный банк. А что касается уровня требований, то все-таки надо четко понимать: есть базовый и есть повышенный, и задания единого государственного экзамена составлены именно на двух уровнях, на базовом и повышенном, поэтому повышенный уровень, конечно, ориентирован не на часовое изучение, а на изучение в большом объеме времени.

Оксана Галькевич: Анна Ивановна, допущен ли сейчас к изучению предмета химии учебник Хомченко, спрашивают ваши коллеги.

Дмитрий Добротин: Это пособие для поступающих в ВУЗы.

Анна Костромина: Это не учебник, это пособие.

Дмитрий Добротин: Как такового учебника такого нет.

Анна Костромина: Это как дополнение для желающих.

Оксана Галькевич: Спасибо, уважаемые друзья, спасибо нашим экспертам. Всем, кто готовится к экзамену по химии, мы желаем удачи, уважаемые друзья.

Юрий Коваленко: Ни пуха ни пера, точнее.

Оксана Галькевич: А 100-балльников, кстати, много?

Дмитрий Добротин: 0.49%, это достаточно много. 0.5% – это хороший показатель, один из самых высоких по сдаваемым предметам.

Оксана Галькевич: Очень здорово. Желаю вам 1% 100-балльников в ЕГЭ этого года.

Дмитрий Добротин: Спасибо.

Анна Костромина: Спасибо.

Оксана Галькевич: У нас в студии сегодня, уважаемые друзья, были Дмитрий Юрьевич Добротин, руководитель Федеральной комиссии разработчиков контрольно-измерительных материалов ЕГЭ и ОГЭ, и Анна Ивановна Костромина, учитель химии московской школы №1028. Спасибо большое.

Юрий Коваленко: Спасибо.

Оксана Галькевич: А мы не прощаемся с вами, вернемся через несколько минут.

Дмитрий Добротин: Спасибо вам.

Анна Костромина: Спасибо.

ЕГЭ по химии 2018: демоверсия, обзор изменений

ЕГЭ по химии в 2018 году претерпит некоторые изменения. Часть из них носит существенный характер. О них и пойдет речь в данной статье. В частности, будет рассказано о том, как меняется структура экзамена и что появилось в нем новое.

Изменения и их смысл

ЕГЭ по химии 2018 сдают как обучающиеся в 11 классе, так и выпускники колледжей и техникумов, желающих продолжить обучение в университете, а значит, подготовка к ЕГЭ по химии должна проходит тщательно.

Главные изменения в ЕГЭ по химии  связаны с отсутствием части с выбором ответа. Теперь даже самые простые задания требуют короткого письменного ответа. Это исключает возможности угадать ответ и мотивирует подготовить тесты более качественно.

Новости ЕГЭ информируют о том, что суммарное количество заданий ЕГЭ по химии возросло до 35 за счет добавления шестого во вторую  часть. Введены задания с общим контекстом. К примеру, в таком виде представлены № 30 и №31. Здесь проверяется усвоения материала по теме «Реакции окислительно-восстановительные» и «реакции ионного обмена».

С учетом уровня сложности претерпевает изменения шкала оценивания. Далее разберем, как именно поменялась шкала в некоторых заданиях.

Оценивание некоторых задач

Задание под номером 9, имеющее повышенный уровень сложности, ориентируется на проверку знаний по теме, касающихся химических свойств веществ неорганического вида. В нем надо установить соответствие между веществами и продуктами получившейся реакции. Максимальная оценка за него 2 балла. В №21 базового уровня проверяются знания по теме окислительно-восстановительных реакций. Нужно установить соответствие между компонентами двух множеств. Правильное выполнение принесет экзаменуемому 1 балл

№26 базового уровня проверяет учащегося по таким темам, как экспериментальные основы химии и представления о важнейших веществах, получаемых промышленными методами. Оценка задания также составляет 1 балл.

№ 30 и 31 относят к высокому уровню сложности. Каждое из них оценивают в 2 балла, направлено на знание реакций в веществах.

Задачи 2 части предполагают развернутый ответ и проверку от 2 до 5 элементов. В зависимости от количества требуемых элементов оценка за каждую задачу варьируется от 2 до 5 баллов. Перечислим задания этой части:

№ ЗаданияМаксимальный балл
302
312
324
335
344
353

Добавлено новое 9 и 31 задание, упрощенное 10, которое является в ЕГЭ по химии 2018 года двадцать первым.

Все изменения направлены на улучшение знаний учащегося и процедуры проверки и его умению ориентироваться в химию. Первичный балл максимально составляет 60, для вузов его пересчитывают по 100-бальной отметке.

Как будет проходить экзамен

Экзаменуемому предоставляется 210 минут на выполнение всей работы ЕГЭ 2018 по химии. На каждое задание отводится определенное время:

  • Базовый уровень сложности – выполняется в течение 1-3 минут;
  • На повышенный уровень сложности можно потратить до 7 минут;
  • Последние задания высокого уровня сложности 2 части выполняются до 15 минут.

Каждая версия экзаменационной работы хранится в КИМах и вскрывается только в присутствии учащихся. Помимо самих заданий, дополнительно выдается таблица химических элементов, растворимости веществ в воде, а также электрохимические напряжения в металлах. Разрешается применять непрограммируемый калькулятор. Черновики выдаются только по требованию учащегося.

Демоверсия экзамена

На официальном портале фипи уже появился демонстрационный вариант ЕГЭ по химии. Он приблизительно похоже на те, которые будут на официальном экзамене. Каждое задание расписано подробно, с указанием количества баллов. В конце приведены правильные ответы и расписано, за что дается максимальное количество баллов. Демоверсия ЕГЭ по химии будет полезна всем, кто собирается сдавать этот экзамен, особенно при поступлении на химический и медицинский факультеты, а также в строительные институты на архитектуру.

Оценка статьи:

Загрузка…

Поделиться с друзьями:

Изменений к экзаменам IB в мае 2021 г. (научная версия)

Вся следующая информация основана на следующем IB pdf , в котором излагаются изменения в экзаменационной сессии в мае 2021 года из-за нарушения COVID. Это сообщение в блоге будет обновлено, когда IB опубликует дополнительную информацию.

Из-за многомесячной стагнации обучения из-за переноса преподавания в онлайн в ответ на пандемию COVID-19 IB решила перенести экзамен на май 2021 года.По большинству предметов IB удалил или изменил части выпускных экзаменов, чтобы уменьшить объем контента, который студентам необходимо будет пройти. Что это значит для вас как студента? Давайте посмотрим, какие изменения внесла IB в науку!

Химия

Документ 3: удален

Проект группы 4: требование удалено

Биология

Документ 3: удален

Проект группы 4: требование снято

Физика

Документ 3: удален

Проект группы 4: требование удалено

Итак, статья 3 удаляется для всех, независимо от того, какую из основных наук вы изучаете, и независимо от того, занимаетесь ли вы SL или HL! Мы в Lanterna думаем, что это одно из изменений экзаменационных сессий в мае 2021 года, которое поможет вам больше всего.Экзамен 3 заведомо сложен в рамках естественных наук, прежде всего потому, что так много студентов вынуждены учить их самостоятельно, так как у них нет времени на их изучение в классе!

С удалением задания 3 вы почти наверняка сможете пройти весь учебный план со своим учителем в классе (будь то очно или виртуально), гарантируя, что в день экзамена не будет ничего, что вы не видели раньше. !

Единственный недостаток, на который вы можете претендовать с этим изменением, заключается в том, что в результате удаления одного документа IA, Paper 1 и Paper 2 теперь будут иметь относительно больший вес.Однако мы думаем, что время, которое вы выиграете от того, что вам не придется рассматривать этот вариант, с лихвой компенсирует любые недостатки!

IB еще не опубликовало официальную информацию о новом весе IA, Paper 1 и Paper 2 после удаления Paper 3, но как только мы узнаем, мы обновим это сообщение в блоге, так что следите за обновлениями!

Подготовка к экзамену AP Chemistry 2021

22 марта 2021 г., Даниэль Блосс

Поскольку COVID-19 продолжает поражать школы по всей стране, Совет колледжей в этом году будет предлагать цифровые экзамены Advanced Placement в дополнение к обычным бумажным экзаменам.

Экзамены AP Chemistry в этом году вернулись к своей стандартной полной продолжительности: 3 часа в обычное время и 4,5 часа в расширенном. Есть некоторые существенные различия между бумажными и цифровыми экзаменами для теста AP Chemistry в этом году, в первую очередь то, что цифровая версия будет включать гораздо больше вопросов с множественным выбором (MC) и меньше вопросов с бесплатными ответами. Читайте даты тестирования, подробности о двух форматах тестов и советы по обучению, и знайте, что Marks Education поможет вам подготовиться к этим необычным экзаменам AP и наилучшим образом воспользоваться тем, что мы знаем и можем предоставить вам. подготовлен к тесту!

ФОРМАТ ЭКЗАМЕНА
И ВАЖНАЯ ИНФОРМАЦИЯ

Даты испытаний

В этом году экзамен AP Chemistry будет сдан в три даты:

  • Пятница, 7 мая, 8:00 по местному времени — экзамен по бумаге, в школе
  • Вторник, 25 мая, 8:00 по местному времени — экзамен по бумаге, в школе
  • Чт 10 июня, 12:00 EDT — Цифровой экзамен, в школе или дома

Бумажный экзамен

Раздел I: множественный выбор
  • 60 вопросов, 1 час 30 минут (50% оценки за экзамен)

Раздел II: Бесплатный ответ
  • 7 вопросов, 1 час 45 минут (50% оценки за экзамен)

Цифровой экзамен

Раздел I: множественный выбор
  • 60 вопросов, 1 час 30 минут (50% оценки за экзамен)

Раздел II: Множественный выбор и бесплатный ответ
  • MC: 40 вопросов, 60 минут
  • ЧТО: 3 вопроса, 45 минут
  • Итого: 1 час 45 минут (50% оценки за экзамен)

Два других примечательных различия между бумажными и цифровыми тестами заключаются в том, что 1) учащимся не разрешается отправлять рукописные ответы для FRQ в цифровом тесте, и 2) учащиеся не могут перемещаться между вопросами на цифровом тесте.Этот второй момент более важен, так как он означает, что учащиеся должны будут завершить и представить свой ответ на каждый вопрос, прежде чем переходить к следующему вопросу, и не смогут вернуться к вопросам после завершения раздела.


Содержание экзамена

Ниже приведена таблица, в которой показано распределение тем за предыдущие годы для раздела «Множественный выбор». Темы FRQ из года в год меняются гораздо больше.

Подготовка к экзамену AP Chemistry

К счастью, есть много отличного материала, который студенты могут использовать для подготовки к экзаменам AP, особенно FRQ. Ниже приведен список советов по подготовке к настоящему тесту.

  1. Откройте домашнюю страницу курса AP Chemistry в AP Classroom на веб-сайте College Board. Для каждого устройства есть наборы задач MC и FRQ, и это отличные инструменты для проверки.Учитель должен их разблокировать, поэтому, если они все еще заблокированы, попросите учителя открыть их, чтобы начать просмотр.
  2. Узнайте, какие темы охватываются на экзамене . Кроме того, документ AP Chemistry Course and Exam Description содержит обзор всех затронутых тем на стр. 22-24, а затем гораздо более подробную информацию на стр. 27-187. Рассмотрите книгу Barron’s или Princeton Review или поработайте с преподавателем для обзора конкретной темы.
  3. Используйте прошлые FRQ и их рекомендации по оценке , которые College Board предоставляет на странице AP Chem (ссылки ниже).Это одни из лучших ресурсов, которые у вас есть. Попытайтесь ответить на эти вопросы после тщательного анализа содержания и проработайте их так, как если бы вы отвечали на них в реальном тесте. Сравните свои ответы с опубликованными правилами выставления оценок, чтобы приблизительно определить, сколько очков вы заработаете. Тогда изучите правильные ответы!

2019 FRQ

ЧТ за 2014-2018 гг .:

до 2014 г .:

FRQ за период до 2014 года можно найти в формате PDF с помощью простого поиска в Интернете с указанием года, например «AP Chemistry Free Response pdf 2012».

  1. Узнай свою реакцию! Вести список для просмотра и знать как общие формы, так и типичные специфические реакции. Например:

Общие: оксиды неметаллов плюс водные оксикислоты

Специфический: CO 2 + H 2 O → H 2 CO 3

  1. Знать периодическую таблицу внутри и снаружи . Это ваш основной инструмент и справочник, который содержит так много информации о каждом элементе, от его массы до его электронной структуры, его химических и физических свойств, просто благодаря его расположению на столе.Также убедитесь, что понимаете периодические тенденции и стоящие за ними силы.

Советы по сдаче экзаменов
  1. Будьте аккуратны и внимательны в своей практической работе , насколько вам нужно будет пройти настоящий тест. Поскольку это открытые вопросы со свободным ответом, они оцениваются людьми, которым необходимо уметь читать и понимать ваши ответы и следить за вашими расчетами. Сделайте так, чтобы им было как можно проще начислять вам все возможные баллы. Также не забудьте указать единицы измерения!
  2. Обратите внимание на сиг-фиги! По FRQ по химии учащимся дается грация в одну значащую цифру в любом направлении. Это означает, что если в ответе на задачу должно быть 3 сиг-инжира, ответы с 2, 3 или 4 сиг-инжиром будут считаться правильными.Однако стоит обратить на это внимание и убедиться, что округление выполняется правильно, чтобы не потерять очки!
  3. Попробуйте каждую часть FRQ. Большинство вопросов состоит из нескольких частей, и за неправильные ответы нет штрафных санкций, поэтому обязательно указывайте что-нибудь для каждой части каждого вопроса, даже если это предположение. А если вы угадываете, постарайтесь использовать свою логику и рассуждения, чтобы приблизиться к правильному ответу, и включите связанные термины и словарный запас, чтобы попытаться получить частичную оценку.

Воспользуйтесь доступными ресурсами, чтобы полностью подготовиться к экзамену AP Chem в этом году и получить 5 баллов!


Marks Education предлагает широкий спектр услуг по консультированию и наставничеству для студентов, желающих поступить в средние школы, колледжи и аспирантуру.
Подробнее об изменениях в экзаменах AP 2020 читайте в нашем блоге по теме здесь.
Если вам нужна помощь при подготовке к экзамену AP Chemistry, ознакомьтесь с биографиями наших опытных преподавателей AP Chemistry здесь: Dan Bloss , Dan Hertz , Ian Perez , Элли Ку

Комментарии закрыты.

MH CET 2018: Изменения в программе, схеме экзаменов, процессе подачи заявок, датах экзаменов и других деталях

Управление технического образования, Махараштра объявило о некоторых изменениях в экзамене Maharashtra Common Entrance Test (MHT CET), которые будут введены с 2018-19. DTE Maharashtra сообщило, что эти изменения были внесены, чтобы сделать уровень экзамена таким же, как экзамен JEE.

Чувство непредсказуемости и нервозности велико для экзаменов MHT CET 2018, когда эти изменения будут реализованы впервые.Посредством этого теста совет допускает кандидатов на курсы BE, BTech, BPharma и PharmD в штате.

Экзамен проводится один раз в год, при этом схема экзаменов может незначительно варьироваться в зависимости от курса, на который кандидат поступает. До сих пор он полностью основывался на программе 12 класса Совета HSC (Махараштра).

MHT CET Изменения внесены:

— Ранее MHT CET полностью основывался на программе класса 12 Совета HSC штата Махараштра.Однако теперь ученики должны будут изучить и программу 11-го класса. Около 20% вопросов экзамена будут основаны на программе 11 класса, а остальные — на программе 12 класса.

— Вопросы, основанные на физике, химии и математике, станут немного сложнее и будут более прикладными. С другой стороны, вопросы биологии будут на одном уровне с экзаменом NEET.

Схема экзамена MHT CET 2017:

Экзамен MHT CET 2017 состоял из 250 вопросов, которые были разделены на 3 работы.Работа 1 содержала вопросы по физике и химии, работа 2 включала вопросы по ботанике и зоологии, а работа 3 содержала вопросы по математике. Каждый доклад был рассчитан на 90 минут, и каждый предмет содержал 50 вопросов и весил 100 баллов за предмет. Математика имела вес в 100 баллов.

Не было необходимости посещать все три доклада. Кандидаты на инженерные факультеты должны были фигурировать только в листах 1 и 3, тогда как кандидаты, ищущие медицинские курсы, должны были фигурировать только в листах 1 и 2.При поступлении на курсы аптек работа 1 была обязательной, и кандидаты могли выбрать любую или обе работы из работ 2 и 3.

Схема экзамена MHT CET 2018:

Основное изменение, внесенное в схему экзамена, заключается в том, что вопросительный лист теперь будет содержать вопросы из программы 11 класса. Общие условия остаются прежними. Экзамен по-прежнему будет разделен на три части, в общей сложности 250 вопросов. Кандидаты по-прежнему должны будут выглядеть так же, как они были в 2017 году, в соответствии с выбранным ими курсом.

Работа 1 будет включать вопросы по математике, работа 2 по физике и химии и работа 3 по ботанике и зоологии. Однако 50 вопросов по каждому предмету будут разделены — 10 вопросов из программы класса 11 и 40 вопросов из программы класса 12. В общей сложности 50 вопросов из 250 теперь будут из программы 11 класса. Каждый предмет будет иметь вес в 50 баллов, за исключением математики, которая будет иметь вес в 100 баллов.

Кандидаты на инженерные факультеты теперь должны будут появиться на экзаменах 1 и 2, кандидаты в медицину должны будут появиться на экзаменах 2 и 3, а для кандидатов в аптеки бумага 2 обязательна.

Программа MHT CET 2017:

1. Физика

Круговые движения, гравитация, вращательные движения, колебания, упругость, поверхностное натяжение, стационарные волны, волновое движение, кинетическая теория газа и излучения, волновая теория света, интерференция и дифракция, электростатика, электричество тока, магнитные эффекты электрического тока, магнетизм , Электромагнитная индукция, электроны и фотоны, атомы, молекулы и ядра и системы связи

2.Химия

Твердое состояние, растворы и коллигативные свойства, химическая термодинамика, электрохимия, химическая кинетика, общие принципы и процессы выделения элементов, блочные элементы P, блочные элементы D и F, координационные соединения, производные галогена, алканы и арены, спирты, Фенолы и эфирные спирты, альдегиды, кетоны и карбоновые кислоты, органические соединения, содержащие азот, биомолекулы, полимеры и химия в повседневной жизни.

3.Биология (ботаника и зоология)

Генетика и эволюция, Биотехнология и ее приложения, Биология и благополучие человека, Физиология растений, Размножение организмов, Экология и окружающая среда, Структура и деление клеток, Разнообразие в живом мире, Физиология человека, Структурная организация животных и растений.

4. Математика

Математическая логика, матрицы, непрерывность, дифференцирование, применение производной, неопределенная интеграция, определенные интегралы, соотношение, пропорция и партнерство, комиссионные, брокерские услуги и скидки, страхование и аннуитет, демография, двумерные данные и корреляция, регрессионный анализ, случайные переменные и вероятность Распределение.

Дополнительные темы для обсуждения на MHT CET 2018:

1. Физика

Сила, измерения, лучевая оптика, преломление света, скаляры и векторы, трение в твердых телах и жидкостях

2. Химия

Основные понятия химии, состояния вещества (газы и жидкости), химия поверхности, природа химической связи, водород, окислительно-восстановительные реакции, элементы s-блока, основные принципы органической химии.

3.Математика

Тригонометрические функции и тригонометрические функции составных углов, факторизация, прямая, окружность и коники, множества, отношения, функции, вероятность, последовательность и серии

4. Биология

Биохимия клетки, водные отношения и минеральное питание, развитие и рост растений, ткани животных, питание человека и дыхание человека.

Право на участие в программе MHT CET:

— Должен сдать класс 12 согласно HSC Board of Maharashtra и быть гражданином Индии.
— Для прохождения инженерного курса абитуриенты должны сдать 12 класс по физике, математике, английскому языку и химии / биологии / биотехнологии / профессиональным предметам с оценками не менее 50 процентов (общая категория).
— Для поступления на фармацевтический курс абитуриенты должны сдать 12 класс по физике, химии, английскому языку и один из оставшихся предметов (как указано выше) с оценками не менее 45 процентов.
— Для поступления на смежные курсы здоровья студенты должны сдать 12 класс по физике, химии, биологии и английскому языку.

Ни один экзамен не является сложным, если он хорошо подготовлен. Всего наилучшего!

— Автор: Devyani Madan, Collegedunia.com

Экзамен AP Chemistry 2021

В 2020 году в ответ на сбой, вызванный COVID-19, Совет колледжа изменил экзамены AP, чтобы они были короче, проводились онлайн, охватывали меньше материала и имели другой формат, чем предыдущие тесты. Но тысячи студентов столкнулись с техническими проблемами и были вынуждены пересдать экзамены.

В этом году Совет колледжей устанавливает 3 даты тестирования, начиная с начала мая, для каждого экзамена AP с вариантами тестирования в школе и дома, а также традиционного и цифрового тестирования.

«Несмотря на то, что варианты экзаменов в этом году выглядят иначе, чем в прошлые годы, этот график максимизирует возможности для учащихся AP в различных ситуациях тестировать и получать кредиты в колледже и размещение», — говорится в сообщении College Board.

Экзамен AP Chemistry на 2021 год будет полноценным экзаменом, который учащиеся могут сдавать в школе и / или дома по традиции (т.е., бумага и карандаш) или в цифровом виде, в зависимости от даты тестирования.

Вот ключевые изменения, которые вам нужно знать, чтобы хорошо сдать экзамен AP Chemistry.

Получу ли я кредит AP за экзамен AP 2021 по химии?

Студенты, сдающие экзамен AP Chemistry 2021 года, будут иметь право на зачет колледжа.

Как и в предыдущие годы, студент должен набрать 3, 4 или 5 баллов, чтобы иметь право на зачет колледжа.

Как долго длится экзамен AP Chemistry в 2021 году?

Экзамен AP Chemistry 2021 будет длиться 3 часа 15 минут.Студенты могут сдавать его в школе или дома в зависимости от даты тестирования (подробности ниже).

Какая дата сдачи экзамена AP Chemistry в 2021 году?

Совет колледжей предлагает в этом году 3 даты сдачи экзамена AP по химии. Время и место зависят от даты.

Бумага и карандаш
Дата Время Местоположение Метод
Пятница, 7 мая 2021 г. 8 утра по местному времени В школе
Вторник, 25 мая 2021 г. 8:00 по местному времени В школе Бумага и карандаш
Четверг, 10 июня 2021 г. 12:00 EDT В школе и дома Цифровой

Однако каждая школа может выбрать наиболее подходящий вариант экзамена.

Что будет проверено на экзамене AP Chemistry 2021?

Экзамен AP Chemistry 2021 будет проверять студентов по всему содержанию курса независимо от того, какую версию экзамена они сдают — традиционный или цифровой — поэтому будьте готовы ответить на вопросы по этим темам:

  • Блок 1: Атомные структуры и свойства
  • Блок 2: Структура и свойства молекулярных и ионных соединений
  • Блок 3: Межмолекулярные силы и свойства
  • Блок 4: Химические реакции
  • Блок 5 : Кинетика
  • Блок 6: Термодинамика
  • Блок 7: Равновесие
  • Блок 8: Кислоты и основания
  • Блок 9: Приложения термодинамики

Что такое формат экзамена AP Chemistry на 2021 год?

Хотя и бумажный экзамен AP 2021 по химии, и цифровой экзамен длится 3 часа 15 минут, их формат отличается.

Экзамен paper будет разделен поровну на 2 части: вопросы с множественным выбором и вопросы со свободным ответом. Каждый раздел оценивается в 50% баллов на экзамене. У вас будет 1 час 30 минут, чтобы ответить на 60 вопросов с несколькими вариантами ответов, и 1 час 45 минут, чтобы ответить на 7 вопросов с бесплатными ответами.

На экзамене digital , однако, будет больше вопросов с несколькими вариантами ответов и меньше вопросов с бесплатными ответами. У вас будет 1 час 30 минут, чтобы ответить на 60 вопросов с несколькими вариантами ответов в первом разделе (как в бумажном экзамене), что составляет 50 процентов баллов за экзамен.Во втором разделе у вас будет 1 час, чтобы ответить еще на 40 вопросов с несколькими вариантами ответов, и 45 минут, чтобы ответить на 3 вопроса с бесплатными ответами.

Чтобы получить бесплатные примеры вопросов с бесплатными ответами, проверьте здесь актуальные вопросы, заданные на экзамене AP Chemistry 2019 года, и здесь, чтобы ознакомиться с вопросами, заданными на экзамене 2018 года.

С начала апреля вы также можете практиковаться в вопросах с множественным выбором и свободными ответами в приложении цифрового экзамена.

Есть ли разница между традиционным и цифровым экзаменом AP Chemistry на 2021 год?

Как бумажный экзамен AP по химии 2021 года, так и цифровой экзамен позволят вам использовать научный или графический калькулятор, такой как графический калькулятор TI-84 от Texas Instruments, для ответов на вопросы бесплатно.

Однако, если вы сдаете цифровой экзамен AP Chemistry, вам следует знать несколько вещей. Во-первых, на цифровом экзамене будет больше вопросов с несколькими вариантами ответов (как описано выше). Во-вторых, вы будете использовать клавиатуру, чтобы отвечать на вопросы со свободным ответом. Вы сможете вводить свои ответы, даже если они содержат символы, так как они включены в приложение для цифрового экзамена. В-третьих, вам не нужно рисовать или строить графики как часть вашего ответа. В-четвертых, вы не можете отправлять ответы, написанные от руки или сфотографированные (в отличие от прошлого года, когда Совет колледжа подготовил условия для сдачи экзамена AP Chemistry 2020 года).

Какое устройство мне следует использовать для цифровой сдачи экзамена AP Chemistry 2021?

То, что вам потребуется для сдачи цифрового экзамена, будет зависеть от того, будете ли вы сдавать цифровой экзамен в школе или дома. В любом случае вам следует заранее определить, какое устройство лучше всего подходит для вас.

Если вы сдаетесь цифровой экзамен в школе , вам понадобится ноутбук (Mac, Windows или управляемый школой Chromebook) со встроенной камерой.Это может быть персональный или школьный компьютер. Если это предоставленный школой компьютер, его нельзя использовать совместно между учащимися после завершения настройки экзамена.

Чтобы сдать цифровой экзамен дома , вам понадобится портативный или настольный компьютер (Mac, Windows или Chromebook под управлением учебного заведения) со встроенной или подключенной камерой. Это может быть персональный или школьный компьютер. Если это предоставленный школой компьютер, его нельзя использовать совместно между учащимися после завершения настройки экзамена.

Независимо от типа устройства или того, кому оно принадлежит, убедитесь, что ваш компьютер полностью заряжен в день экзамена, чтобы его хватило на весь экзамен. Если вы сдаете цифровой экзамен дома, убедитесь, что ваша настройка включает в себя зарядку во время экзамена, если это необходимо.

Вам также следует установить приложение для цифрового тестирования не позднее, чем за день до экзамена.

Вам, конечно же, понадобится надежный доступ к Интернету, если вы будете сдавать цифровой экзамен.

Предлагает ли Совет колледжа какие-либо бесплатные курсы обзора экзамена AP по химии?

Совет колледжей предлагает студентам бесплатные ресурсы AP.

Уроки в прямом эфире и записи, проведенные учителями AP со всей страны, доступны на канале AP YouTube. Кроме того, с 19 по 29 апреля учащиеся будут иметь доступ к сеансам AP Daily: Live Review, которые помогут им ознакомиться с содержанием курса и навыками перед экзаменом. Вы можете записаться на сеансы просмотра в реальном времени здесь.

Какие еще варианты подготовки к экзамену AP Chemistry 2021?

Если у вас еще нет обзорной книги AP Chemistry, попробуйте TUN’s Textbook Save Engine, чтобы сравнить цены и получить лучшие предложения.

Есть также онлайн-ресурсы, которые вы можете использовать, чтобы помочь вам подготовиться к экзамену AP Chemistry 2021 года.

Академия Хана

В сотрудничестве с College Board, Khan Academy предлагает официальный курс AP Chemistry с бесплатными обучающими видео, статьями и практическими упражнениями, созданными нынешними и бывшими преподавателями AP. Кроме того, Khan Academy предлагает бесплатные экзаменационные навыки и стратегии.

Princeton Обзор

Princeton Review предлагает 6-часовой курс AP Cram для экзамена AP по химии.Курс охватывает темы статистического вывода, оценки, вероятности и т. Д. Эксперт Princeton Review поможет вам подготовиться к экзамену.

Курс, к которому прилагается книга для подготовки Princeton Review, можно заказать онлайн за 299 долларов. На данный момент доступно 3 расписания курсов повышения квалификации.

  • 24 и 25 апреля, 12:00 до 15:00 (Восточное время)
  • 26 и 28 апреля, 19:00 до 22:00 (Восточное время)
  • 1 и 2 мая, 15:00 до 6:00 р.м. (По восточному времени)

Princeton Review также предлагает частные уроки, доступные как лично, так и онлайн, за плату от 167 долларов в час. Если вы решите выбрать этот индивидуальный вариант, опытные наставники будут работать с вами, чтобы «составить план, поставить цели и превзойти их». Princeton Review гарантирует, что если вы не будете удовлетворены на 100%, вам подберут другого репетитора и вы получите следующий урок бесплатно.

Если вы сдали более одного экзамена AP в 2021 году, ознакомьтесь с обзором экзамена AP TUN, чтобы узнать о других пересмотренных экзаменах AP.

Руководство к экзамену AP Chemistry

Знаете ли вы разницу между эндотермическими и экзотермическими процессами? Сможете ли вы сбалансировать окислительно-восстановительную реакцию? Экзамен по химии AP ® проверяет темы и навыки, обсуждаемые в вашем продвинутом курсе химии. Если вы наберете достаточно высокий балл, ваша оценка AP Chemistry может принести вам кредит в колледже!

Ознакомьтесь с нашим Руководством по химии AP, чтобы получить важную информацию, необходимую для экзамена:

Что на экзамене по химии?

Экзамен AP Chemistry длится 3 часа 15 минут.Экзамен состоит из двух разделов: раздела с несколькими вариантами ответов и раздела с бесплатными ответами.

Сроки

Количество вопросов

% баллов за экзамен

Раздел 1

90 минут

60 вопросов с несколькими вариантами ответов

* Использование калькулятора запрещено.

50%

Раздел 2

105 минут

3 длинных вопроса с бесплатными ответами
4 коротких вопроса с бесплатными ответами

* Использование калькулятора разрешено.

50%

Примечание: Совет колледжа еще не объявил, будет ли цифровая версия экзамена AP Chemistry 2021 года на место для будущих экзаменов.Для получения обновленной информации о цифровом тесте и его формате посетите страницу AP Chemistry на веб-сайте College Board.

AP Chem Обзор: темы

Темы экзамена AP по химии сгруппированы по девяти разделам.

  • Раздел 1: Структура и свойства атома. Темы могут включать: моли и молярную массу, масс-спектроскопию элементов, элементный состав чистых веществ, состав смесей, атомную структуру и электронную конфигурацию, фотоэлектронную спектроскопию, периодические тенденции, валентные электроны и ионные соединения.

  • Раздел 2: Структура и свойства молекулярных и ионных соединений. Темы могут включать: типы химических связей, внутримолекулярную силу и потенциальную энергию, структуру ионных твердых тел, структуру металлов и сплавов, диаграммы Льюиса, резонанс и формальный заряд, VSEPR и гибридизацию связей.

  • Раздел 3: Межмолекулярные силы и свойства. Темы могут включать: межмолекулярные силы, твердые тела, жидкости и газы, кинетическую молекулярную теорию, растворы и смеси, фотоэлектрический эффект.

  • Блок 4: Химические реакции. Темы могут включать: введение в реакции, чистые ионные уравнения, представления реакций, физические и химические изменения, стехиометрию, типы химических реакций.

  • Раздел 5: Кинетика. Темы могут включать: скорость реакции, введение в закон скорости, элементарные реакции, модель столкновений, введение в механизмы реакции, профиль энергии многоступенчатой ​​реакции и катализ.

  • Раздел 6: Термодинамика.Темы могут включать: эндотермические и экзотермические процессы, теплопередачу и тепловое равновесие, теплоемкость и калориметрию, энергию фазовых переходов, введение в энтальпию реакции, энтальпию образования, закон Гесса.

  • Блок 7: Равновесие. Темы могут включать: введение в равновесие, расчет константы равновесия, расчет равновесных концентраций, введение в принцип Ле Шателье, введение в равновесие растворимости, pH и растворимость, свободную энергию растворения.

  • Блок 8: Кислоты и основания. Темы могут включать: введение в кислоты и основания, pH и pOH сильных кислот и оснований, кислотно-основные реакции и буферы, молекулярную структуру кислот и оснований, pH и pK и , свойства буферов.

  • Раздел 9: Приложения термодинамики. Темы могут включать: введение в энтропию, свободную энергию Гиббса и термодинамическую благоприятность, термодинамический и кинетический контроль, свободную энергию и равновесие, гальванические (гальванические) и электролитные элементы, электролиз и закон Фарадея.

AP Chem Типы вопросов

Множественный выбор

Разделы с множественным выбором AP Chem содержат отдельные вопросы или наборы вопросов, в которых учащимся предоставляется стимул или набор данных и ряд связанных вопросов. Вам не разрешается использовать калькулятор во время этого раздела экзамена.

Оценка: Этот раздел оценивается компьютером.

Бесплатный ответ

Вопросы с бесплатными ответами AP Chemistry оценивают вашу способность:

  • Объяснять, анализировать и интерпретировать модели и представления химических свойств или явлений
  • Разработка экспериментов и процедур для проверки предсказания или теории
  • Представляйте данные и химические явления в виде графиков и диаграмм
  • Решать задачи с помощью математических соотношений
  • Сделать или оправдать научное утверждение и подкрепить его доказательствами и / или аргументацией

К этому разделу экзамена допускаются четырехфункциональные, научные или графические калькуляторы.

Оценка: Наибольшую часть баллов вы получите в этом разделе за демонстрацию процесса, который привел вас к ответу. Оценщики частично доверяют, когда вы показываете им, что знаете, что делаете. Ответы оцениваются комитетом учителей средних школ и колледжей и оцениваются в соответствии со стандартом, установленным в начале оценочного периода главными консультантами факультета.

AP Chem Equation Sheet

Таблицы, содержащие часто используемые уравнения и константы в химии, предоставляются вместе с экзаменом AP Chemistry и могут использоваться как для вопросов с множественным выбором, так и для вопросов с произвольным ответом.Тестируемым также предоставляется периодическая таблица элементов. Ознакомьтесь с курсом AP Chem и описанием экзамена от College Board для получения дополнительной информации.

Как интерпретировать результаты химического анализа AP

Оценки AP указываются от 1 до 5. Колледжи обычно ищут 4 или 5 на экзамене AP Chemistry, но некоторые могут предоставить зачетные 3 балла (ознакомьтесь с нашим обзором того, как заработать зачет AP). Каждый тест имеет изогнутую форму, поэтому баллы меняются от года к году. Вот как студенты сдали экзамен AP Chemistry в мае 2020 года:

Оценка

Значение

Доля испытуемых

5

Высоко квалифицированный

10.6%

4

Хорошо квалифицированный

18,6%

3

Квалифицированный

26,9%

2

Возможно квалифицированный

24%

1

Нет рекомендаций

29.9%

Источник: College Board

Как подготовиться?

классов AP — это здорово, но для многих студентов этого недостаточно! Для тщательного изучения содержания и стратегии AP Chemistry выберите вариант подготовки AP, который лучше всего подходит для ваших целей и стиля обучения.


The Staff of Princeton Review
Более 35 лет учащиеся и семьи доверяют изданию Princeton Review помочь им попасть в школу своей мечты.Мы помогаем учащимся преуспеть в старшей школе и за ее пределами, предоставляя им ресурсы для получения более высоких оценок, более высоких результатов тестов и более сильных заявлений в колледж. Следуйте за нами в Twitter: @ThePrincetonRev.

Химическая олимпиада


Открыта регистрация на Олимпиаду 2021 года. Щелкните ссылку ниже, чтобы зарегистрировать студентов.

Программа олимпиады по химии в Юго-Восточном Мичигане.

Национальная химическая олимпиада США (USNCO) — это программа для школьников-химиков. USNCO спонсируется на национальном уровне Американским химическим обществом (ACS) и организовывается на местном уровне Детройтской секцией ACS. Учащиеся из девяти округов, обслуживаемых Детройтской секцией, имеют право участвовать.

The конечной целью USNCO является выбор четырех члены команды Соединенных Штатов по международной химии Олимпиада (МХО) состоится практически в этом году.Процесс отбора начинается с экзамена в местной секции в марте. На юго-востоке Мичигана одиннадцать лучших студентов на экзамене местной секции (после поправки на национальные правила) называются номинантами Детройтской секции. Номинанты и девять призеров получат денежные призы в размере 150 и 75 долларов соответственно. Наряду с примерно тысячей студентов по всей стране кандидаты сдают (виртуальный) национальный экзамен в конце апреля. Двадцатка лучших на национальном экзамене будет приглашена в учебный лагерь олимпиады в Академии ВВС в Колорадо-Спрингс.Затем после десяти напряженных и изнурительных дней в учебном лагере выбираются четыре члена американской команды.
Онлайн-регистрация для сдачи экзамена Детройтской местной секции на USNCO
.
Экзамен местной секции на олимпиаде по химии 2021 года будет проводиться в субботу, 27 марта. Онлайн-регистрация откроется 4 января 2021 года. На экзамен местной секции могут подавать не более двадцати учеников в каждой школе. Учителя средних школ нажмите ЗДЕСЬ , чтобы зарегистрировать до пятнадцати учеников и пяти их заместителей для сдачи экзамена USNCO в Детройте в 2021 году.Если вы предпочитаете зарегистрироваться paper, вы можете скачать ПЕЧАТНУЮ РЕГИСТРАЦИОННУЮ ФОРМУ . Кроме того, вы можете связаться с доктором Марком ДеКэмпом по адресу [email protected] и регистрационные материалы будут отправлены вам по почте. Подтверждение будет отправлено по электронной почте после получения регистрационных материалов. полученный. Крайний срок приема регистрационных материалов понедельник, г. 1 февраля г.

Изменения на 2021 год. Из-за пандемии экзамен местной секции будет проводиться удаленно.Студенты сдают экзамен через Moodle в субботу, 27 марта. Подробная информация о входе в систему будет доступна в ближайшее время. Экзамен будет проводиться учителями местной средней школы.

Кто должен участвовать? Приглашаются все учащиеся-химики средней школы, но лучше всего подготовятся к экзамену местной секции лица, которые закончили или в настоящее время записываются на курс химии AP.

Взимается ли участие? Участие в местном секционном экзамене на олимпиаде по химии бесплатно.Студенты должны принести свои калькуляторы (в соответствии с правилами конкурса). Будут предоставлены листы с ответами Scantron, карандаши № 2 и макулатура. Каждый участник получит значок олимпиады.

Национальные и местные правила олимпиады по химии Обратите внимание, что национальные правила заменяют правила местных секций.

Расписание Олимпиады 2021

  • Начало января. Регистрационные материалы отправлены учителям .Если вас нет в списке рассылки Комитета по образованию Детройтской секции, свяжитесь с доктором Марком ДеКэмпом по адресу [email protected], и вам будут отправлены материалы.
  • Понедельник, 4 января. Открывается онлайн-регистрация.
  • Понедельник, 1 февраля. Крайний срок регистрации
  • Суббота, 27 марта. Экзамен секции Экзамен местной секции Детройта будет проводиться онлайн Суббота, 27 марта с 15:00 до 16:30.Учителя несут ответственность за обеспечение безопасности экзаменов.
  • Начало апреля. Двадцать пять лучших студентов на экзамене местной секции будут приглашены ответить на десять дополнительных вопросов с несколькими вариантами ответов и вопрос-эссе (бесплатный ответ), чтобы разорвать связи и выбрать окончательный список кандидатов и участников, занявших второе место.
  • Суббота, 17 апреля, и суббота, 24 апреля. Национальный экзамен , части I и II Национального экзамена будут проводиться удаленно.
  • Начало мая. Участники учебного лагеря идентифицировали человек.
  • 24 июля . Международная химическая олимпиада начинается. Самый последний экзамен (2019 г.) Местной секции Детройта и ключ ответов можно найти здесь. Экзамен местной секции состоит из семидесяти вопросов с несколькими вариантами ответов. и вопрос со свободным ответом. Часть экзамена с несколькими вариантами ответов основана на тесте, созданном ACS. с десятью дополнительными вопросами.Используется вопрос со свободным ответом с целью разрыва связей. Сто шестьдесят учеников из двадцати четырех школ сдали экзамен по местной секции в марте прошлого года. Средний балл по экзамену 2019 был прав на 35,9 (из 70) с высоким баллом 62.

    Номинанты на олимпиаду 2019, школы-участники и школы-участницы
    Щелкните ЗДЕСЬ , чтобы просмотреть список номинантов, школ-призеров и школ-участниц на олимпиаде по химии в Детройте в 2019 году. Также доступна гистограмма, показывающая распределение баллов на экзамене местной секции.

    Детройтское отделение Архивы USNCO
    • Представители Детройта в учебном лагере USNCO, 1985-2020 гг.
      Секция Детройта участвует в программе USNCO с 1985 года. За двадцать четыре из последних тридцати шести лет Детройт направил по крайней мере один ученик в учебный лагерь при Военно-воздушной академии. В 2019 году Яджван Раван (средняя школа Черчилля) представлял Детройтскую секцию в учебном лагере. По завершении учебного лагеря Яджван был выбран в команду США, и в итоге он получил золотую медаль на Международной олимпиаде по химии в Париже! Полный список всех участников учебного лагеря из Детройтской секции можно найти ЗДЕСЬ .здесь.
    • Старые вопросы для эссе При выборе номинантов и призеров в Детройте ничья разрывается путем бесплатного ответа на вопрос. Тема вопроса для сочинения обычно своевременна. Вопросы для эссе из последних десяти экзаменов Детройтской местной секции собраны ЗДЕСЬ .
    • D Собственные загружаемые копии общих экзаменов местной секции и прошлых национальных экзаменов можно найти на сайте ACS USNCO .

    Дополнительная информация

    Дополнительную информацию о программе олимпиады по химии можно найти на национальном веб-сайте U ГНКО .

    Фотогалерея с экзаменов местной секции и национальных экзаменов 2019

    Экзамен местной секции

    Сдача экзамена местной секции. Хорошо, это экзамен 2018 года, но каждый год он выглядит одинаково.

    Учителя на встрече SEMCTO 2018, Стив Космас представляет


    Национальный экзамен и ужин с наградами

    Кандидаты, сдающие часть I Национального экзамена

    Практическая лаборатория


    Будущий обладатель золотой медали Яджван Раван (средняя школа Черчилля) работает над одной из лабораторных задач.

    Основной докладчик профессор Эван Триведи, факультет химии Оклендского университета


    Одиннадцать номинантов после вручения денежных призов. Слева направо: М. Ван, Ким, П. Сони, К. Хва, А. Чжан, В. Наяк, К. Мазел, Т. Лво, А. Релекар, Ю. Раван, Л. Венг (отсутствуют: С. Ганеш)

    Яджван Раван, третий слева, показывает свою золотую медаль после Международной олимпиады по химии.

    Последнее обновление 02.02.20

Январь 2018 Экзамен регента по химии Подробные ответы.

Презентация на тему: «Подробные ответы на экзамен регента по химии, январь 2018 г.» — стенограмма презентации:

1 Подробные ответы на экзамен регента по химии за январь 2018 г.

2

3 Протоны и нейтроны в ядре, электроны летают вне ядра.
Старая модель Далтона называется бильярдным шаром (твердая сфера). Томпсон придумал модель сливового пудинга, в которой электроны погребены в «положительной каше» атома, но она НЕ включена. Следующей была модель Резерфорда атомов с положительным ядром с летающими электронами снаружи, в результате чего объем атома в основном был пустым. Следующей была модель Бора с электронами, летящими по орбитам, или оболочкам. Наконец — но это тоже не входит в ответы, у нас есть современная модель, известная как волновая механическая модель, с электронами на орбиталях, а не орбитах.N, Ne, Mg, Si — это неметаллы, благородный газ, металл и металлоид, по порядку. Группа 15 — это N, P и As, затем Sb и Bi. Все атомы в группе имеют одинаковое количество валентных электронов (вне орбитали), и все они имеют 5 электронов (ВЗГЛЯД).

4 5. Сульфид аммония имеет катион Nh5 + 1 и анион S-2.
5. Сульфид аммония имеет катион Nh5 + 1 и анион S-2. Заряды перекрестно перекрещиваются, чтобы получить (Nh5) 2S, что означает два аммония. за один сульфид.6. Эмпирические формулы — это на самом деле (я имею в виду тупица) математика. Ищите САМЫЙ СНИЖЕННЫЙ, который часто даже не является «настоящим веществом». Nh4 нельзя сокращать. Остальные все можно уменьшить. 7. Температура и плотность — это физические свойства. Нейтроны похожи, их нельзя отличить друг от друга. Химические свойства используются, чтобы отличить химически разные вещества, что и отличает соединения друг от друга. Формула Эмпирическая формула N2O4 NO2 C3H6 Ch3 P4O10 P2O5

5 8.Лед — это вода, h3O, представляет собой молекулярное соединение
8. Лед — это вода, h3O — это молекулярное соединение. В нем нет металлов, поэтому он не ионный (как NaCl). Он бывает однородным (одинаковым во всем), потому что это чистое вещество, но это НЕ смесь. Он не является ни гетерогенным, ни смешанным. 9. CO2 имеет ДВЕ двухполярные ковалентные связи и имеет форму прямой линии, поэтому ИМЕЕТ радиальную симметрию. Это НЕ ПОЛЯРНАЯ МОЛЕКУЛА, и у нее симметричное распределение зарядов — оба атома кислорода получают электроны от углерода, потому что кислород имеет более высокую электроотрицательность.Атомы кислорода ОТРИЦАТЕЛЬНЫ, атом углерода более ПОЛОЖИТЕЛЬНЫ. Радиальная симметрия сохраняет молекулу неполярной. Элементы, которые не вступают в реакцию, являются БЛАГОДАРНЫМИ ГАЗАМИ. Благородный газ здесь — гелий.

6 11. Классический вопрос. Когда образуются связи, выделяется энергия
11. Классический вопрос! Когда образуются связи, высвобождается энергия. Здесь 2 атома кислорода (которые по отдельности нестабильны) связываются вместе (двойные неполярные ковалентные связи) и становятся более стабильными. Энергия высвобождается по мере образования связей.12. Притягивание электронов от других атомов называется электроотрицательностью. Его описал Линус Полинг. Отсутствие разницы в значениях электроотрицательности означает неполярную связь. Чем больше разница, тем больше полярность связи. Фтор имеет наивысшее значение электроотрицательности 4,0, и это как произвольная шкала (числа не имеют значения, они просто ранжируются), так и относительная шкала, все атомы измерены ОТНОСИТЕЛЬНО к одному атому (F). Вся медь имеет одинаковую плотность (она постоянна для каждого вещества.Вся медь имеет одинаковые химические свойства (все они связываются одинаково). Только элементы не могут быть разрушены химическим изменением, поэтому выбирайте Вольфрам или W.

7 KMT — это теория, объясняющая, как могут существовать газы и как они остаются газами. Из этих вариантов только: идеальные газы находятся в случайном, постоянном прямолинейном движении. Без передачи энергии столкновения не являются упругими.Регулярные геометрические узоры — это сплошные. Небольшие расстояния — это жидкости или твердые тела, частицы газа действительно далеко друг от друга. Ах! Вопрос о гипотезе Авогадро. Проведите тонны молекулярной математики или запомните следующее: равные объемы разных газов при одинаковой температуре и давлении имеют одинаковое количество молей и частиц. Итак, 6 литров Cl2 (G) на STP содержат такое же количество молекул, как 6 литров любого другого газа в мире на STP. Точно так же, как влюбленность, реакция не может произойти, если у вас не будет столкновений с достаточной энергией и правильной ориентацией.Диаграммы потенциальной энергии показывают ПОТОК ЭНЕРГИИ в химической реакции с течением времени.

8 19. Катализаторы не влияют на частоту столкновений.
19. Катализаторы не влияют на частоту столкновений. Скорее они УМЕНЬШАЮТ ЭНЕРГИЮ АКТИВАЦИИ или ПРЕДЛАГАЮТ АЛЬТЕРНАТИВНЫЙ ПУТЬ для протекания реакции (быстрее). АЛКИНЫ на столе Q. ПОСМОТРИТЕ на него сейчас, чтобы знать, что у вас там есть. Это Cnh3n-2. Перенос электронов может происходить только в результате образования ионов (что НЕ является выбором) или ОКИСЛЕНИЕ — ВОССТАНОВЛЕНИЕ (окислительно-восстановительный потенциал).Электрохимия. 22. Аноды… вместо этого подумайте так: LEO — это КРАСНАЯ КОШКА (что означает восстановление происходит на катоде. Итак, аноды делают другое, на аноде происходит окисление. 23. Гальванические элементы — это батареи, химия производит электричество. Электролитическим элементам требуется электричество для создания окислительно-восстановительного потенциала (гальваники).

9 кислоты Аррениуса перечислены в таблице K, СМОТРЕТЬ СЕЙЧАС
кислоты Аррениуса перечислены в таблице K, СМОТРИТЕ СЕЙЧАС.Кислоты Аррениуса выделяют ионы водорода в растворе, чем больше H + 1, тем они более кислые. HBr нет в списке, но он похож на HCl. NaBr не является кислотой. NaOH и Nh4 являются основаниями (таблица L). Вы проделали эту лабораторную работу во время кислотно-основного титрования. Вспомните, как база сделала раствор ярко-розовым! Это был фенолфталеин (таблица M). Это классическая нейтрализация кислотным основанием: кислота + основание = соль и вода. Это та самая «глупая» теория под названием Брэнстед Лоури. Вы должны запомнить эту реакцию: Nh4 + HOH → NH OH-1, что означает, что аммиак и вода образуют катион аммония и анионы гидроксида. Аммиак является основанием, потому что он ПРИНИМАЕТ ион водорода.Вода — ОСНОВА, потому что Жертвует ион водорода. Кислоты «отдают H + 1» в этой тупой теории.

10 28. Переход от одного атома к другому — это действительно круто, и мы узнали об этих безумных вещах только в конце года. Это даже не химия, это ЯДЕРНАЯ ХИМИЯ. Это трансмутация. Присоединение и замещение — это органические реакции, восстановление — окислительно-восстановительные. Нестабильное ядро ​​может испускать различное излучение.Излучение представляет собой либо чистую энергию (гамма-излучение), либо форму частиц, таких как альфа или бета, позитроны, протоны или нейтроны (посмотрите сейчас на таблицу O). Его также называют РАДИОАКТИВНЫМ РАСПАДОМ. Атомные электростанции, когда они работают безупречно, создают небольшие проблемы, за исключением их долговременных радиоактивных отходов (с которыми люди еще не знают, что делать). Если они сломаются, чего почти следовало ожидать, они могут быть действительно опасными, выбрасывая все виды токсичных радиоактивных отходов в воздух, воду и почву.Это особенно плохо.

11 Все атомы (или ионы), которые имеют 7 протонов (положительные) и 10 электронов (отрицательные), имеют заряд -3. Нейтроны никогда не имеют зарядов, они включены, чтобы сбить вас с толку. ВСЕ анионы азота имеют 7 протонов и 10 электронов. В возбужденных состояниях электроны находятся в более высоких конфигурациях, чем нормальные. Все атомы имеют конфигурацию ОСНОВНОГО СОСТОЯНИЯ в периодической таблице. Сравните эти. У атома с 5 электронами (рифмуется с бором!) 2-3 (если вы посмотрите) означает, что он возбужден.

12 33. Это самый сложный вопрос из всех.
33. Это самый сложный вопрос из всех! Я ненавижу это, потому что это сложно, просто быть трудным. Мы поступили бы так: (10.01 а.е.м.) (. 199) = PLUS (11.01 amu) (. 801) = is: amu. Выбор 1! Они НЕ меняли сначала проценты на десятичные числа. Чтобы приспособиться к этой глупости, они обманывают вас, деля все уравнение на. Это делает моль правильно, но только сумасшедшие делают математические вычисления таким образом.34. Неметаллы находятся с правой стороны лестницы (плюс H). Атомы, которые получают электроны, чтобы стать анионами (чтобы стать изоэлектрическими по отношению к благородному газу), — это O, S и Se. Они образуют анионы O-2, S-2 и Se-2. Они ВСЕ В ГРУППЕ 16. Посмотрите на свою периодическую таблицу.

13 Значения электроотрицательности
35. Вы можете проверить каждую тенденцию, чтобы понять это. Сделайте диаграмму. Или будь ленивым, и сделай ошибку Не будь ленивым.Вариант 2, 1-я энергия ионизации уменьшается, это единственный правильный ответ. атом Значения электроотрицательности 1-я энергия ионизации Валентные электроны Число орбиталей F 4,0 1681 кДж 7 2 Cl 3,2 1251 кДж 3 Br 3,0 1140 кДж 4 I 2,7 1008 кДж 5 Групповая тенденция к снижению константа увеличения

14 29 г KF 1 X = 0,50 моль KF (2 SF) 1 моль KF 58 г KF
36. Выполните математические вычисления. Сначала определите молярную массу → KF K 1 x 39 = 39 г ЗАТЕМ… F 1 x 19 = 19 г / моль Выполните преобразование… Полярность связи основана на разнице в электроотрицательности. Чем больше разница, тем полярнее связь. Составьте диаграмму. (не поленитесь) Наибольшая разница с H — O 29 г KF 1 X = 0.50 моль KF (2 SF) 1 моль KF 58 г Выбор KF Разность атомов 1 C 2,6 O 3,4 0,8 2 H 2,2 1,2 3 N 3,0 0,4 4 S 2,6

15 M = = = 1,00 M Количество молей растворенного вещества Литры раствора
38. Таблица F показывает нам, какие ионные соединения являются водными или не водными (нерастворимыми) в воде. (посмотрите на таблицу F), он не ионизируется и не диссоциирует в воде.Если оставить в стороне причудливые символы, «НАИМЕНЬШАЯ» растворимость для наших целей НЕ РАСТВОРИМОСТЬ. Ненавижу их и за этот каверзный вопрос. Напишите формулу еще раз. Делать математику. У вас есть 2 образца воды одинакового размера, один холоднее (10 ° C), другой горячее (20 ° C) ПО СРАВНЕНИЮ С ОБРАЗОМ 1, ОБРАЗЕЦ 2 СОДЕРЖИТ… молекулы с более высокой средней кинетической энергией, с более высокой средней скоростью, с БОЛЬШЕ ТЕПЛА энергия. Кол-во моль растворенного вещества Литры раствора 0,500 моль Литры M = = = M

16 41.Мы любим диаграммы частиц! Ищем здесь ХИМИЧЕСКИЕ ИЗМЕНЕНИЯ.
Вариант 1 показывает, что газ превращается в твердое тело, что является осаждением (противоположно сублимации. Вариант 2 показывает твердое тело → газ, которое является сублимацией. Вариант 3 не показывает ничего. У вас есть 3 молекулы газа слева и те же три молекулы основного вещества на справа. Это может быть CO (G) или HCl (G). Вариант 4 показывает, что 2 молекулы становятся двумя разными двухатомными молекулами. Скорее всего, 2 молекулы воды становятся 2 молекулами водорода (черные точки = H) и одна молекула кислорода (белые точки = O )

17 42.Посмотрите на таблицу H, но сначала посмотрите на таблицу K (кислоты). Найдите эту формулу, которая представляет собой Ch4COOH, которая является органической формулой для уксусной кислоты. Вы также можете назвать это этановой кислотой. Найдите 90 ° C, и вы четко увидите, что оно должно быть 40 кПа. Еще одна сложная вещь с этой таблицей … Температурные ячейки = 5 ° C на ящик. Убедитесь, что вы нашли ПРАВУЮ точку на графике. Взгляните на это прямо сейчас. Не поленитесь. 43. Упорядоченные частицы означают ТВЕРДОЕ. В жидкостях есть подвижные частицы (которые немного липкие, но не прилипают).Газы — это просто дикие частицы, случайное движение по прямой, далеко друг от друга, постоянно движущееся, вероятно, уже в Бруклине!

18 44. Разыгрывается 101,3 кПа и 298 К
44. Разыгрывается кПа и 298 К. Какое название к таблице I? (посмотрите сейчас!) Найдите реакцию с C3H6 (G) в качестве продукта и найдите ∆H: это кДж / моль Ding ding: эта реакция дает ОДИН моль пропена.В этом вопросе говорится: 2 МОЛИ х 2 = кДж (они просто сказали, черт возьми, их можно снова возненавидеть!) 45. Изомеры имеют одинаковую химическую формулу, но построены по-разному, они имеют другую структурную форму. У них тоже разные свойства, они НЕ одинаковые. Создайте диаграмму, не поленитесь. ПОДСЧИТАЙТЕ АТОМЫ СИНИМУ сейчас. Выбор Проблемы 1 метанол и метанол Ch4OH против Ch3O X разные формулы 2 пропановая кислота и пентановая кислота 3 атома углерода против 5 атомов углерода X разное количество атомов 3 1-пропанол и 2-пропонол Ch3OHCh3Ch4 и Ch4Ch3OHCh4 ТОЧНЫЕ АТОМЫ, разные формы 4 1 хлорпропан и 2 бромпропановый хлор vs.бром X различные галогениды

19 Мое любимое, вот как я делал банановое масло (сложный эфир, называемый метилгексаноатом) в колледже органической химии в Кортленде! Этерификация — это реакция между органической кислотой и спиртом. 47. Нитрат калия (KNO3) находится в таблице F. Он ионный, и вы видите, что он также водный. Ионные соединения, которые находятся в воде, диссоциируют или ионизируют и образуют рыхлые подвижные ионы в растворе. Они проводят электричество, что является определением электролита KNO3 (S) → K + 1 (AQ) + NO3-1 (AQ) 48.Шкала pH представляет собой логарифмическую шкалу (шкала экспонент), и каждое изменение целого числа = 10-кратное изменение. Она находится в диапазоне от 0 до 14. Меньшие числа, ниже 7, являются кислотами. Если кислотный pH изменяется на целое число (ниже), теперь это 10-кратное изменение. более кислый Если кислотный pH изменяется на целое число (выше), он не становится в 10 раз менее кислым Здесь кислотные ионы уменьшаются в 10 раз, pH должен увеличиваться на 1

20 Время от жидкости до твердого только от D до E, или от 7 до 9 минут.Выбор 4
49. На этой кривой охлаждения верхняя (самая горячая) часть графика — это пар, а правая нижняя — сплошной лед. Я обозначил для вас баллы. Соотнесите их со временем, и все готово. A-B — это только газ. B-C — фазовый переход G → L, конденсация. C-D — только охлаждающая жидкость. D-E — фазовый переход. Вариант 4

21 год 50.Это горение метана, и оно тоже экзотермическое.
50. Это горение метана, и оно тоже экзотермическое. Реагентами являются метан и кислород, для более эффективных столкновений вы можете увеличить их температуру (не выбор), увеличить площадь их поверхности (на самом деле это не относится к газам) или вы можете увеличить их концентрацию (что правильно). Вы можете попробовать катализатор, чтобы ускорить его, но он НЕ влияет на столкновения. Реагенты → Продукция

22 51.Преобразование точки плавления ртути в градусы Цельсия
51. Преобразование точки плавления ртути в градусы Цельсия. Найдите температуру в таблице S, это 234 Кельвина. Напишите формулу и выполните математические вычисления: K = C → K = C → C = -39,0 ° C 52. Точечная диаграмма Льюиса для монофторида водорода (HF) H имеет одну точку (1 электрон), F — всего 7 точек в сочетании, F — октет. Не ионный, скобок здесь нет. 53. Числовая установка (не решайте, но вы можете) q = mHV = (102,3 г) (2260 Дж / г)

23 Кислотное основание Chem, Таблица М.Этот раствор, 0
Acid base Chem, Table M. Этот раствор, 0,10 M Nh4 является слабым основанием, которое будет иметь pH более 7, но, вероятно, меньше, чем Все растворы с pH более 4,4 в метиловом оранжевом становятся желтыми. Это буквы А и D. Посмотрите сейчас на эту картинку. Смесь внизу содержит много чего. Некоторые из них могут быть A, D X, Z и т. Д. Мы не знаем, но мы можем согласовать выбор со смесью. A и D есть.

24 С точки зрения электронов И энергетических состояний, как создается свет, излучаемый возбужденными атомами.
Обычно атомы имеют электроны в основном состоянии (самые низкие уровни энергии). Когда они получают энергию, электроны могут поглощать определенное количество энергии и подпрыгивать на более высокие орбитали, чем обычные. Это возбужденное состояние. Когда эти электроны излучают эту недавно поглощенную энергию, это и есть спектры. Когда они излучают эту энергию, они могут вернуться к орбиталям основного состояния или уровням энергии, на которых они обычно живут. Спектр ИЗЛУЧАЕТСЯ, когда энергия высвобождается.

25 57.Вряд ли это химия, это чтение таблицы и знание разницы между физическими и химическими свойствами. Укажите химическое свойство ЙОДА. Правильный выбор: образует ионные связи с активными металлами или образует ионные связи или реагирует с металлами. Взгляните на периодическую таблицу сейчас: Основное состояние Rb — Основное состояние I — Запись: Rb имеет атомный радиус 215 пм, в то время как I имеет атомный радиус 136 пм. Электропроводность зависит (в основном) от того, являются ли элементы металлами или нет.Металлы проводят электричество, а неметаллы — нет. Рубидий — металл, он хорошо проводит электричество. Йод — неметаллическое твердое вещество, оно вообще не проводит электричество.

26 год 60. Сбалансируйте это уравнение: в буклете для ответов оно выглядит следующим образом: ____ P4 (S) + ____ Cl2 (G) → ____ PCl3 (L) + энергия Если вам нужна цифра «1», вы можете не указывать ее или написать в графе «1». 1 ”, здесь оба в порядке. Ответ: P4 (S) + 6 Cl2 (G) → 4 PCl3 (L) + энергия или: P4 (S) + 6 Cl2 (G) → 4 PCl3 (L) + энергия 61.Объясните, почему это синтез: напишите: два элемента объединяются, образуя одно соединение, или два реагента образуют один продукт, или два вещества вступают в реакцию с образованием одного продукта.

27 62. Численная установка процентного содержания PCl3 по массе. Вы тоже можете решить эту проблему. PCl3 P 1 x 32 = г PX 100% =% Cl 3 x 35 = 105 г / моль Cl X 100% = 76,6%%. требуется для точки, вы должны сделать всю работу, чтобы убедиться, что Nuclear Chem! Что касается нейтронов и протонов: почему U-238 и U-234 — разные изотопы урана? Оба эти атома представляют собой уран, но с разной массой.Все атомы урана имеют протоны (они №92 в периодической таблице). Массы — это сумма протонов + нейтронов. У них разное количество нейтронов. U-238 имеет 92 протона и 146 нейтронов (= 238). U-234 имеет 92 протона и всего 142 нейтрона (= 234) 32 136

28 год Распадающиеся частицы (справочную информацию см. В таблице O
Распадающиеся частицы (справочную информацию см. В таблице О. Th-234 распадается (излучает радиоактивность и превращается в атом другого типа)) Вау (да, вау) Сделайте простые математические вычисления, верхние числа, а затем нижние числа Решите для недостающей части.Стрелка представляет собой знак =. «Математика» вверху: 234 = XX = 0 «Математика» внизу: 90 = XX = -1. Частица в таблице O, которую мы ищем, не имеет массы (верхнее число) и заряд -1 (нижнее число, которое может быть только БЕТА-ЧАСТИЦЕЙ, которая выглядит так: 234 90 Th X + 234 91 Па 0 -1 e или

29 65. Проблемы с периодом полураспада требуют, чтобы вы нарисовали t-диаграмму, иначе вы, вероятно, ошибетесь. ВЕСЬ ПОЛОВИНА ВОСЬМОЙ ШЕСТНАДЦАТЫЙ Время Half Life HL HL HL Чтобы иметь только одну шестнадцатую от начальной массы, вы должны пройти четыре периода полураспада. Для этого материала, Т-234, период полураспада указан в днях.Посчитайте: 4 X дней = дней Эмпирические формулы (тупой!) C2Cl4 — это составная формула, эмпирическая формула: CCl2

30 2NaHCO3 (S) + тепло → Na2CO3 (S) + h30 (L) + CO2 (G) 67
2NaHCO3 (S) + тепло → Na2CO3 (S) + h30 (L) + CO2 (G) Многоатомный ион в твердым продуктом является анион CO3-2, называемый карбонатным анионом. 68. Тепловой поток между воздухом в духовке и жидким тестом: воздух в духовке намного горячее, тепло переходит от горячего к холодному (холод — это просто недостаток тепла).Тепловые потоки из воздуха в жидкое тесто. Жидкая вода. Потенциальная энергия по сравнению с паром. Вода. Потенциальная энергия. Жидкая вода имеет более низкую потенциальную энергию, чем пар.

31 год Значимые цифры давления на поверхности озера 104,0 кПа — 4 SF
Рассчитайте объем пузыря на дне озера… Начните с закона комбинированного газа, затем заполните пробелы. Используйте свои единицы! Масса пузырька воздуха на поверхности: используйте формулу плотности P1 V1 T1 P2 V2 T2 (618.3 кПа) (V1) 282 K (104,0 кПа) (2,5 мл) 293 K = = Масса Объем D = г / мл 1 Масса 2,5 мл Масса = г / мл X 2,5 мл = Масса = граммы или 3 x 10-3 г знак равно

32 73. Укажите доказательства того, что прямая реакция является экзотермической.
73. Укажите доказательства того, что прямая реакция является экзотермической. Энергия является продуктом прямой реакции, которая является экзотермической. 74. Сравните скорости прямой / обратной реакции. В динамическом равновесии скорость прямой реакции равна скорости обратной.75. Какое напряжение делает больше коричневого газа (делает обратный сдвиг)? Уменьшите давление, добавьте энергии, добавьте тепла, увеличьте температуру или увеличьте объем емкости (что снижает давление).

33 76. Сравните межмолекулярные силы в NO2 и N2
76. Сравните межмолекулярные силы в NO2 и N2? NO2 — полярная молекула, обладающая дипольным притяжением. Они сильнее, чем силы электронной дисперсии N2.У NO2 более сильный IMF, чем у N2. 77. Степень окисления N в NO Эта молекула должна иметь степени окисления, которые в сумме равны нулю. Оба атома кислорода имеют степень окисления -2, в то время как азот имеет много степеней окисления на выбор, выполните «математику» X + -4 = X для азота будет +4

34 78. Моль ионов Mg + 2, образующихся при взаимодействии 2, 5 моль Zn + 2
78. Моль ионов Mg + 2, образующихся при взаимодействии 2.5 моль Zn + 2 реагируют. Это окислительно-восстановительный потенциал (электрохимия). Из приведенного уравнения должно быть ясно, что на каждую реакцию Zn + 2 вы получаете один Mg + 2. Следовательно, если реагируют 2,5 моля Zn + 2, то образуется 2,5 моля Mg + 2. 79. Пористый барьер — это «солевой мостик», который позволяет потоку ионов между половинными ячейками, который сохраняет нейтральность растворов, а аккумулятор (гальванический элемент) может продолжать работу. Mg более реактивен, чем Zn (см. Таблицу J). . Вы могли бы сказать, что Zn менее активен, чем Mg, или даже если Mg выше в таблице J, чем Zn. Запишите сбалансированную половину реакции окисления в этой ячейке: Mg ° → Mg e-

35 год 82.Почему углеводород в уравнении 1 ненасыщен
82. Почему углеводород в уравнении 1 ненасыщен? C2h5 + Cl2 → C2h5Cl C2h5 ненасыщен, потому что это алкен (см. Таблицу O), или между двумя атомами углерода существует двойная связь, что означает, что они могут содержать больше атомов водорода — если они связаны одинарной связью, как это делает алкан C2H6. в этой реакции — C2h5Cl2. Он классифицируется как галогенуглерод или галогенид. Возможно, вам удастся обойтись галогеналканом.

Добавить комментарий

Ваш адрес email не будет опубликован. Обязательные поля помечены *